Sei sulla pagina 1di 140

Applied Mathematics II

CHAPTER 1

REAL SEQUENCES
Introduction: Let’s start off this chapter with a discussion of just what a
sequence is! A sequence is nothing more than a list of numbers written in a
specific order. The list may or may not have an infinite number of terms in
them although we will be dealing exclusively with infinite sequences.
Sequences are useful way of talking about infinite sets of numbers {a n } when

there is one number a n for each natural number n. What calculus contributes

to this concept is the notion of limit; that is, we can ask if the values in our
sequence are approaching a specific value as the natural number n goes to
infinity. Evaluating these limits can be done very similarly to evaluating limits
of functions at infinity except now our independent variable is only taking
natural number values instead of real values. In particular, we have analogous
properties and theorems, such as the limit of a sum is the sum of the limits (as
long as all of the limits are defined), and the Squeeze Theorem. Finally, the
Monotonic Sequence and bounded sequences Theorems are deep fact about the
structure of the real number system; know it well.

Objectives: At the end of this chapter, the student is expected to:


 Define Real sequence
 Differentiate monotonic sequences
 Define bounded sequence.
 Define convergent and divergent sequences.
 Check whether a given real sequence is convergent or divergent.
 Explain conditions for convergence of a sequence.
 Apply divergence test to identify divergent sequences.
 Use convergence properties to evaluate limit of a sequence.

1
Applied Mathematics II

Lesson1.1 and 1.2

Lesson title: Convergence of Sequences

What is a sequence?
? When do we say that a sequence is convergent?

Introduction: In this section we will discuss sequences, convergence of


sequences and some theorems about limits of sequences.

Objectives: At the end of this lesson, the student is expected to:


 Define a sequence of real numbers.
 Describe definition of convergent sequence.
 Use ε − N definition to show convergence of sequences.
 Apply Squeeze or Sandwich Theorem to find out convergence of a
sequence whenever necessary.

Definitions and Notations


Informally speaking, the term “sequence” in mathematics is used to describe an
unending succession of numbers. For instance
1, 2, 3, 4, K
2, 4, 6, 8, K
1 1 1
1, − , , − , K
2 4 8
1, − 1, 1, − 1, K
In each case dotes are used to suggest that the sequence continues indefinitely,
following the obvious pattern. The numbers in the sequence are called terms of
the sequence. The terms may be described according to the position they
occupy. Thus a sequence has a first term, a second term, a third term and so
forth.
Consider the following infinite set of numbers

2
Applied Mathematics II

 1 1 1 1 1 
A = 1, , , , , K , , K
 2 3 4 5 n 
1
That is, the set A consists of all numbers of the form , where n is a positive
n
integer. There is another way to describe the set A . That is, we define a function
1
f by the rule f (n) = , where the domain of f is the set of positive integers.
n
Then the set A is precisely the set of values taken by the function f . In general
we have the following definition.
Definition: A sequence of real numbers is a function whose domain is the set of
all non-negative integers greater than or equal to a given integer (usually 0 or
1). The values taken by the function are called terms of the sequence.
Notation: Generally, sequence terms are denoted as follows
a1 - first term

a2 - second term
a 3 - third term

a n - n th term

M
There are varieties of ways of denoting a sequence. Each of the following is
equivalent ways of denoting a sequence

{a1, a2 ,K, an , an +1,K} , {an } , {an }∞n =1 .


Usually ( but not always) the sequence that arises in practice have a
recognizable pattern and can be described by a formula.
Example 1. Find a formula for a n in each of the following cases:

 1 1 1  1
(i) 1, , , , K , a n = for all n ∈ N ,
 2 3 4  n

(ii) {1 , − 1 , 1 , − 1 , K} , a n = (− 1)n +1 for all n∈ N ,

3
Applied Mathematics II

 1 3 7 15  2n − 1
(iii)  , , , , K , a n = n for all n ∈ N ,
 2 4 8 16  2
(iv) {2 , 2 , 2 , 0 , 0 ,0 , 0 , 0 , 0 , K}, a n = 2 if n ≤ 3 , a n = 0 if n ≥ 4 .

Activity 1. Find a formula for the general term a n of the given sequence

assuming that the pattern of the first few terms continues.


a) {1 , 4 , 7 , 10 , K}

b) {0 , 2 , 0 , 2 , 0 , 2 , K}

2 3 4 5 
c)  , − , , − , K
3 5 7 9 
Activity 2. Suppose the number of bacteria in a culture is growing
exponentially, with a doubling time of 10 hours. Suppose also that there are
initially 1000 bacteria in the culture. Find a formula for the number a n of

bacteria in the culture after n hours.


Example 2. Write down the first few terms of each of the following sequences.

 n + 1
a)  2 
 n n =1

 (− 1)n +1 
b)  
 2 n  n =0

Solution:
(a) To get the first few sequence terms here all we need to do is plug in
values of n into the formula given and we’ll get the sequence terms.

 n + 1  3 4 5 6 
 2  = 2 , , , , ,K .
 n n =1  4 9 16 25 
(b) This is similar to the first one. The main difference is that this sequence
doesn’t start at n = 1 .

 (− 1)n +1   1 1 1 1 
  = − 1 , , − , , − , K .
 2 n
 n =0  2 4 8 16 

Note that the terms in this sequence alternate in signs. Sequences of this kind
are sometimes called alternating sequence.

4
Applied Mathematics II

Convergence of sequences
A sequence {a n } may have different values a n for each value of n . It may also

happen that as n increases, the different a n ’s tends to cluster around some

fixed number. In this section our main focus will be on the behavior of
sequences as n increases.

 n  n
Consider the sequence   ; the terms of the sequence an = are
 n + 1 n =1 n +1
approaching 1 as n becomes large. In fact, the difference
n 1
1− =
n +1 n +1
can be made as small as we like by taking n sufficiently large. We indicate this
by writing
n
lim =1
n →∞ n + 1

In general, the notation


lim an = L
n →∞

means that the terms of the sequence {a n } can be made arbitrarily close to L

by taking n sufficiently large.


Definition: A sequence {a n } is said to have the limit L , written as

lim an = L ,
n →∞

if for any ε > 0 , there is a natural number N such that for every n > N , we have
an − L < ε .

This definition is illustrated by Fig1.1 in which the terms a1 , a 2 , a 3 , K are plotted

on a number line. No matter how small an interval (L − ε , L + ε ) is chosen, there


exists N such that all terms of the sequence from a N +1 onwards must lie in that

interval.

5
Applied Mathematics II

Fig1.1

a1 a3 a6 a8 a N +1 a N +2 a9 a7 a5 a4 a2
| • • • • ( • • • ) • • • • •
0 L −ε L L+ε
Fig1.2

The above two figures show the geometrical interpretation of the ε − N


definition.
Remark 1. Some sequences do not satisfy the above definition. We call such
sequences divergent.
2. Sequences which satisfy the above definition, i.e. L exists and is
finite, are called convergent sequences.
3. The number N depends on ε .
4. We may also replace n > N by n ≥ N without changing the sense of
the definition. That is, they give rise to the equivalent definitions.
Example 3. Prove the following limits by using ε − N definition of a sequence.
1
a) lim =0
n →∞ n

1
b) lim =0
n →∞ 2n

6
Applied Mathematics II

1
Solution(a): Given any ε > 0 , we want to find N such that −0 <ε for n > N ,
n
1 1
i.e., n > for n > N . Choose N to be the smallest integer such that N ≥ . ( N is
ε ε
1 1 1
found now!). When n > N , then n > N ≥ or − 0 < ε . Thus lim = 0 .
ε n n →∞ n

Solution(b): This time we have to show that for any ε > 0 , we can find an integer
N such that
1
− 0 < ε whenever n > N
2n
Now
n
1 1 1
−0 = =  ,
2n 2 n
2
So we need to determine the values of n for which
n
1
  <ε .
 2
We need to solve this inequality for n . Since n is in the exponent, we may use
logarithms to simplify the inequality. Now
n
1
  <ε
 2
if and only if
n
1
ln  < ln ε
2
Equivalently
1
n ln  < ln ε ………….(*)
2
1
And notice that ln  < 0 , so the inequality (*) yields
 2

7
Applied Mathematics II

ln ε
n>
1
ln 
2
ln ε 1
Thus if we set N = , then n − 0 < ε whenever n > N .
1 2
ln 
2
Activity 3. Let an = c ,where c is a constant, for n ≥ 1 .Then show that lim an = c .
n →∞

ln n
Activity 4. Calculate lim .
n →∞ n

n2
Example 4. Use ε − N definition to prove that lim =1.
n →∞ n2 + 1

n2
Solution: Given any ε > 0 , we want to find N such that − 1 < ε for n > N .
n2 + 1

Now

n2 n
2
−1 < ε ⇔ −1 < ε
n +1 n2 + 1

n − n2 + 1 n 2 − (n 2 + 1)
<ε ⇔

n2 + 1 (
n2 + 1 n + n2 + 1 ) <ε

(
1
n2 + 1 n + n2 + 1 ) (
< ε ⇔ n2 + 1 n + n2 + 1 > ) 1
ε

1
Choose N to be the smallest integer such that N ≥ . Then, for n > N ,
ε

( ) (
n2 + 1 n + n2 + 1 > N 2 + 1 N + N 2 + 1 > N ≥ ) 1
ε
or
n2
n2 + 1
− 1 < ε . Thus N is

found and hence the result.


Activity 5. Determine if the following sequences converge or diverge. If the
sequence converges determine its limit.

 3n 2 − 1 
a)  2
10n + 5n  n = 2

8
Applied Mathematics II


 e 2n 
b)  
 n  n =1

 (−1) n 
c)  
 n  n =1

{ }
d) (−1) n

n =0

e) { n + 1 − n }

n =1

Activity 6. Use the ε − N definition to show that ( − 1) n { }



n =1 is divergent.

Theorem1.1: If {a n } has a limit, then the limit is unique.

L−M
Proof: Let L and M be limits of {an } . Suppose that L ≠ M . Choose ε = .
2
Then ε > 0 because L ≠ M . By definition, there exist N 1 and N 2 such that

a n − L < ε for n > N 1 and a n − M < ε for n > N 2 . For n > max{N 1 , N 2 }, we have

L−M
L − M = (L − a n ) + (a n − M ) ≤ L − a n + a n − M < 2ε = 2 = L−M ,
2
⇒ L − M < L − M , which is a contradiction.

Thus L = M .

Activity 7. Use the ε − N definition to show that


5n − 3 5
a) lim =
n →∞ 2n + 5 2
n 1
b) lim =
n →∞ 7n + 6 7
Theorem1.2 (Squeeze or Sandwich Theorem): Given three sequences
{a n }, {bn }, {c n } such that
i) a n ≤ bn ≤ c n for every n and

ii) lim a n = L = lim c n ,


n →∞ n →∞

9
Applied Mathematics II

then lim bn = L .
n →∞

Proof: For any ε > 0 , there exists N 1 and N 2 such that c n − L < ε for n > N 1 and

a n − L < ε for n > N 2 . Let N = max{N 1 , N 2 } . For n > N , we have the following:

From an − L < ε , we get ,

− ε < an − L < ε

⇒ L − ε < an < L + ε and hence L − ε < an .......(1.1)

From cn − L < ε ,we obtain

− ε < cn − L < ε

⇒ L − ε < c n < L + ε and cn < L + ε .........(1.2)

Since a n ≤ bn ≤ c n and (1.1) & (1.2) , it follows that

L − ε < a n ≤ bn ≤ c n < L + ε

⇒ bn − L < ε , ∀n ≥ N .

Therefore, lim bn = L , by definition of limit.,


n →∞

Example 5. Find limits


1 + sin n
a) lim
n →∞ n
n
 n −1 
b) lim  
n →∞ 2 n + 1 

1 + sin n 2 2
Solution (a): Since 0 ≤ ≤ and lim = lim 0 = 0 . Therefore
n n n →∞ n n →∞

1 + sin n
lim = 0 , by Squeeze or Sandwich Theorem.
n →∞ n
n n n
 n −1  1 1
Solution (b): Since 0 ≤   ≤   and lim   = lim 0 = 0 , we have
 2n + 1  2 n →∞ 2  n →∞

n
 n −1 
lim   =0.
n →∞ 2 n + 1 

10
Applied Mathematics II

n!
Example 6. Compute lim .
n →∞ nn

n!
Solution: Observe that 0 < for all n > 0 .
nn
n ! 1 ⋅ 2 ⋅ 3L (n − 1) ⋅ n 1 2 3 n − 1 n 1 n! 1
Next observe that n
= = ⋅ ⋅ L ⋅ < . Hence 0 < n < .
n n ⋅ n ⋅ nLn ⋅ n n n n n n n n n

1 n!
Since lim = 0 , also lim n = 0 by the Squeeze or Sandwich Theorem.
n →∞ n n →∞ n

Activity 8. Using the Squeeze or Sandwich Theorem to show that the sequence

 sin(n) 
a)   is convergent.
 n + cos(n)  n =1

 4n + 3 
b)  2  is convergent.
 4n + 3n + 1
Key terms
• Sequences
• Convergence of sequences
• Limit of sequences
Self Test.
1. Let {an } be the sequence for which a1 = 6 and an +1 = 6 + an for n ≥ 1 .

a) Find the first three terms of the sequence.


b) Show the sequence converges and find its limit.
2. Let a1 and k be any positive real numbers and let {an } be the

1 k
sequence for which a n +1 = ( an + ) for n ≥ 1 .Assuming that this
2 an
sequence converges, find its limit.
3. Evaluate the following limit
a) lim n ln n b) lim ( n + 1 − n )
n →∞ n →∞

11
Applied Mathematics II

1
4. We know that lim = 0 = L . This means according the normal
n →∞ n

definition of limit for every ε > 0 , there exists an integer N such that
1
an − L < ε = −0 <ε when n ≥ N .In each part, find the smallest
n
possible value of N for the given value of ε .
a) ε = 0.5 b) ε = 0.1 c) ε = 0.001

2n
5. Prove that lim =2
n →∞ n + 1

Lesson 1.3

Lesson Title: Convergence of Sequences and Sequences which tend


to ∞ or - ∞.
Introduction: In this section we extend our discussion on the convergence of
sequence further in such a way that evaluating convergence or divergence of a
sequence will be done on the basis of the theorems we had on limits of
functions in Applied Mathematics I.

Objectives: At the end of this lesson, the student is expected to:


 Find appropriate function corresponding to a given sequence to evaluate
limit of the sequence.
 Apply definition of sequences approaching ∞ to verify limits of sequences
approaching ∞..
 Identify sequences converging to ∞.

Sequences which tend to ∞

Definition: {a n } tends to + ∞ if for each positive number k , there is a natural

number N such that

12
Applied Mathematics II

a n > k for all n > N . For such sequences, we write a n → +∞ as n → ∞ or

lim a n = +∞ .
n →∞

Similarly, if for every number M, there is an integer N such that if n≥N,

then an < M , we say that {a n } diverges to − ∞ and is written as lim an = −∞


n →∞

Example1. The following sequences tend to + ∞


n
4
a) a n =  
3

n2 −1
b) a n =
n
Exercise: Show !
Theorem 1.3 (Reciprocal rule): Consider a sequence {a n } .

1
i) If a n > 0 for all n and lim = 0 , then lim a n = +∞ .
n →∞ a n →∞
n

1
ii) If lim a n = ±∞ , then lim = 0.
n →∞ n →∞ a
n

Proof: i) For each positive integer k , there exists N such that

1 1
−0 <
an k

1
for n > N because lim = 0 . Then, for n > N , a n > k because a n > 0 .
n →∞ an

Hence lim a n = +∞
n →∞

ii) Suppose lim a n = +∞ . We want to show that given any ε > 0 there is an
n →∞

1
N such that − 0 < ε for all n > N .
an

1
Indeed we know that for every ε > 0 we can find a positive k such that <ε .
k
And for this k there is an N such that a n > k for all n > N since lim a n = +∞ .
n →∞

13
Applied Mathematics II

1 1
This implies <
an k

1 1
⇒ − 0 < < ε for all n > N
an k

1
Therefore, lim = 0.
n →∞ an

The proof of the other case, when lim a n = −∞ , is left for the reader.
n →∞

1
Example 2. Since lim = 0 , we have lim n = ∞ . Similarly, since lim n = +∞ ,
n →∞ n n →∞ n →∞

1
we have lim = 0.
n →∞ n

Theorem 1.4: Let {a n }∞n=m be a sequence, L a number and f a function

defined on [m, ∞ ) such that f (n) = a n for all n ≥ m .



a) If lim f ( x) = L , then {a n }n =m converges and lim an = L .
x →∞ n→∞


b) If lim f ( x) = +∞ (or lim f ( x) = −∞ ) , then {a n }n =m diverges, and lim a n = +∞ (or
x →∞ x →∞ n →∞

lim a n = −∞ ).
n →∞

Thus lim a n = lim f ( x) .


n →∞ x →∞

Proof: a) First we assume that lim f ( x) = L , and let ε > 0 . Then there is some
x →∞

integer N such that


if x ≥ N , then f ( x) − L < ε

This implies that


if n ≥ N , then a n − L = f (n) − L < ε

So that, lim an = L
n→∞

b) If lim f ( x) = +∞ , then for any M there is an integer N such that


x →∞

if x ≥ N , then f ( x) > M

So that, if n ≥ N , then f (n) = a n > M

14
Applied Mathematics II

which implies by definition that lim a n = +∞ .


n →∞

Similarly the reader can show that lim f ( x) = −∞ implies lim a n = −∞ .


x →∞ n →∞

 n 
Example 3. Show that lim   =1
n →∞ n + 1 

x
Solution: Let f ( x) = , for x ≥ 1 .
x +1
n
Then f (n) = = an , for n ≥ 1
n +1
x 1  n 
Since lim = lim = 1 , we conclude from Theorem 1.4 that lim   =1
x →∞ x +1 x → ∞ 1+1 x n →∞ n
+ 1
1
Example 4. Show that lim = 0 , for r > 0 .
n →∞ nr
1
Solution: Let f ( x) = , for x ≥ 1 .
xr
1 1 1
Then f (n) = r
, for n ≥ 1 , and lim r
= 0 from which follows that lim = 0.
n x →∞ x n →∞ nr
Activity 1. Show that
a) lim n c = 1 for c > 0
n →∞

b) lim (1 + n) 2n
=1
n →∞

n
 1
Example 5. Show that lim 1 +  = e .
n →∞ n
x n
 1  1
Solution: If we let f ( x) = 1 +  for x ≥ 1 , then f (n) = 1 +  for n ≥ 1 .
 x  n
x
 1
But lim f ( x) = lim 1 + 
x →∞ x →∞ x
x x
 1  1
If there is a number b such that lim ln1 +  = b , then lim 1 +  = e b .
x →∞  x x →∞ x
x
 1 ln(1 + 1 x )
But lim ln1 +  = lim x ln(1 + 1 x ) = lim
x →∞  x x →∞ x →∞ 1x

15
Applied Mathematics II

(1 + 1 x )(− 1 x 2 )
= lim = 1 (L’Höpital’s rule)
x→∞ (
− 1 x2 )
x
 1
Hence lim 1 +  = e b = e1 = e
x →∞ x
n
 1
Therefore by theorem 1.4 we get lim 1 +  = e .
n →∞ n
Activity 2. Let r be a real number. Then show that
lim r n = 0 if r < 1 .
n →∞

And lim r n = ∞ if r > 1 .


n →∞

Key terms
• Convergence of sequences
• Sequences which tend to ∞
Self Test.
1. Using definition of limit, verify the following.
3n + 1 n2 + 1
a) lim =3 b) lim n = ∞ c) lim =∞ d) lim − 2n3 = −∞
n →∞ n n →∞ n →∞ n n →∞

2. Evaluate the following limits.


1
n
 1 1 n
 0.05 
a) lim 1 +  b) lim arcsin cos  c) lim ∫ e x dx
n →∞
 n  n →∞
 2 n n →∞
1

n

Lesson 1.4

Lesson Title: Properties of convergent sequences.


Introduction: So far we have been using appropriate functions which can be a
associated to a sequence to determine the convergence of a sequence. But a
sequence by itself is a function so that the limit theorems discussed in Applied
Mathematics I can be used to find out convergence or divergence of sequences.

Objectives: At the end of this lesson, the student is expected to:

16
Applied Mathematics II

 Use convergence properties to calculate limit of a sequence.


 Write sequence as a sum or difference or product or quotient of
convergent sequences whenever possible to verify convergence of a given
sequence.

Properties of convergent sequences.


Next, we will continue our analysis of the convergence and divergence of
sequences. Since sequences are functions, we may add, subtract, multiply, and
divide sequences as we do functions. Rules for computing the limits of
combinations of sequences are analogous to the rules for limits of combination
of functions.
Theorem 1.5: Suppose that the sequences {a n } and {bn } converge to limits L

and M , respectively, and C is a constant. Then


a) lim C = C .
n →∞

b) lim Can = C lim an = CL .


n →∞ n →∞

c) lim (a n + bn ) = lim a n + lim bn = L + M .


n →∞ n →∞ n →∞

d) lim (a n − bn ) = lim a n − lim bn = L − M .


n →∞ n→∞ n→∞

e) lim (a n bn ) = lim a n lim bn = LM .


n →∞ n →∞ n →∞

a  nlim an L
f) lim  n  = →∞ = ( if M ≠ 0) .
n →∞ b
 n  nlim
→∞
bn M

Proof : Left as an exercise.


 n 2 + 3n + 2   1 
Example 1. Find limit of ln 2
 + cos
  .
 2 + 4n + 2n   n

  n 2 + 3n + 2
Solution: lim ln
  1 
 + cos   = lim ln 
( )
 n 2 + 3n + 2 n 2   
 + lim cos 1 
    n →∞
n →∞
  2 + 4n + 2n
2
 (
 n  n→∞  2 + 4n + 2n n )
2 2
  n

 1 + 3 n + 2 n2 
= lim ln 2
 + cos 0 why?

n→ ∞
2 n + 4 n + 2

17
Applied Mathematics II

 1+ 0 + 0  1
= ln  + 1 = ln  + 1 = 1 − ln 2 why?
0 + 0 + 2 2
 240 
Example 2. Find the limit of the sequence  .
 n 
Solution: Since we have already seen that
1
lim =0
n →∞ n

it follows that
240 1
lim = 240 lim = (240)(0) = 0 .
n →∞ n n → ∞ n
Activity 1. Show that if the sequence {a n } diverges and the sequence {bn }

converges, then the sequence {a n + bn } and {a n − bn } both diverge.

Let {a n } be a real sequence. Let L be a real number. To know if an → L , we look

at the difference a n − L and if a n − L → 0 ,then the answer is “yes”.

The question is then how do we know a n − L → 0 . Thus the following is a good

test.
Theorem 1.6: If there is a sequence {bn } such that

(i) bn → 0

(ii) a n − L ≤ bn

Then a n → L

Proof: Given ε > 0 , there is an integer N such that

n > N ⇒ bn < ε

Therefore ∀ n > N , a n − L ≤ bn < ε

∴ an → L .

Activity 2. Prove the following


a) lim c n = 0 for any fixed c where c < 1 .
n →∞

18
Applied Mathematics II

b) lim n n = 1 .
n →∞

1
c) lim cn = 1 for any fixed c > 0 .
n →∞

np
d) lim = 0 for any fixed p and c > 0 .
n →∞ cn
cn
e) lim = 0 for any fixed c > 0 .
n →∞ n !

n
 x
f) lim 1 +  = e x for any fixed x .
n →∞ n

g) lim
(ln n ) p
= o for any fixed k > 0 .
n →∞ nk
Hints for activity 2: For (e) Take a positive integer k such that p-k < 0 and apply
L’Höpital’s rule.

cn
For (f) let a n = and fix an integer M>c. Then notice that
n!
for any n>M,
c ⋅ c ⋅L⋅ c ⋅ c c
0 < an = aM < aM .
n ( n − 1) L ( M +1) n
Key terms
• Convergence of combinations of sequences
Self test.
1. Evaluate the following limits.
1 1 1 1 2n + (−1) n
a) lim ( − ) b) lim ( − 1)(1 + 1 ) c) lim
n →∞ n n +1 n →∞ n n n →∞ e2n
2. Use the ε − δ definition to prove that the statement “ lim (−1) n = 0 ” is false.
n →∞

3. Let f ( x) be a function with a continous derivative. Let x1 , x 2 , K, x n ,K be a

sequence obtained by Newton’s method,


f ( xn )
x n +1 = x n − .
f ' ( xn )

19
Applied Mathematics II

Show that if {x n } converges to L and f ' ( L) ≠ 0 , then L is a solution of the

equation f ( x) = 0 .

Lesson 1.5

Lesson Title: Bounded and Monotonic sequences


Introduction: Some times we may be interested to know just the convergence
or divergence of a sequence without finding the limit of the sequence. In such
cases this section has a great importance in determining convergence or
divergence of sequences.

Objectives: At the end of this lesson, the student will be able to:
 Verify whether a given sequences is bounded or not.
 Apply Test for Divergence to identify divergent sequences.
 Explain conditions for the convergence of a monotonic sequence.

Bounded and Monotonic Sequences

Definition: Let {a n }∞n = m be a sequence.

i) The sequence {a n }∞n = m is bounded above if there is a number M 1 such

that a n ≤ M 1 for every positive integer n ≥ m .

ii) It is bounded below if there is a number M 2 such that M 2 ≤ a n for

every positive integer n ≥ m .


It is bounded if there is a number M such that a n ≤ M for every n ≥ m .

Otherwise we say the sequence is unbounded.

Example 1. The sequence {(− 1) }


n ∞
n =1 has upper bound 1, lower bound -1 and

bound 1.

Example 2. The sequence {2 }


n ∞
n =1 has lower bound 2, but has no upper bound

so it is unbounded.

20
Applied Mathematics II

∞ ∞
Theorem 1.7: If {a n }n = m converges, then {a n }n = m is bounded.

Proof: Suppose {a n }n = m converges, in other words there is a real number L such

that lim a n = L , which implies that there is a natural number N such that
n →∞

if n ≥ N , then a n − L < 1

Therefore,
if n ≥ N , then a n = a n − L + L ≤ a n − L + L < 1 + L

Let M be a number larger than a m , a m+1 , K , a N −1 , and 1 + L

Then a n ≤ M for n ≥ m , and thus the sequence is bounded.

1 1 1
Example 4. Let an =1+ + + L + − ln n for n ≥ 1.
2 3 n
a) Using the definition of natural logarithm, show that
1
ln(n + 1) − ln n ≥ for n ≥1
n +1
b) Using (a), show that the sequence {an }n =1 is decreasing.

c) Using the fact that


n 2 3 n
1 1 1 1
∫1 t ∫1 t ∫2 t
dt = dt + dt + L + ∫n −1 t dt for n ≥ 2 .
Show that
1 1
ln n ≤1 + + L + .
2 n −1
And hence that an ≥ 0 for all n

d) Show that {an }n =1 converges.


1
Solution: (a) Consider the function f ( x) = , on the int erval [n , n + 1] for n ≥ 1 .
x
1
On this interval in particular, the function f ( x) = is decreasing since
x
1
f ' ( x) = − < 0 and by The First Derivative Test.
x2
Thus,

21
Applied Mathematics II

n +1
1 1

n x
dx ≥
n +1
[ n + 1 − n]

1
⇔ ln(n + 1) − ln(n) ≥
n +1

(b) At this stage we have to show that the given sequence is decreasing.
Now we can use the above fact to derive the following result,
 1 1 1 1   1 1 1 
an +1 − an = 1+ + + L + + − ln(n + 1) − 1 + + + L + + − ln(n)
 2 3 n n +1   2 3 n 
1
= ln(n) − ln(n + 1) + ≤0
n +1

Therefore the sequence {an }n =1 is decreasing.


(c) Now let us use the given information to show that the above-
mentioned inequality is true.
Given the interval [1, n] we can break it down into subintervals of unit length.

So, the interval can be written as [1, 2]∪ [2 , 3]∪L∪ [n −1, n] . Knowing that the
1
function f ( x) = is decreasing on [1, n] , the value of the function is greater at
x
the left end point of the subinterval than every other point in the corresponding
subinterval.
Then,
n 2 3 n
1 1 1 1
∫1 t dt = ∫1 t dt + ∫2 t dt +L + n∫−1 t dt
1 1 1
≤ [2 − 1]   + [3 − 2]   + L + [n − (n − 1)]
1 2 n −1
1 1
⇒ ln n ≤1+ + L +
2 n −1
1 1
⇒1 + + L + − ln n ≥ 0
2 n −1
1 1 1
⇒1 + + L + + − ln n ≥ 0
2 n −1 n
⇒ an ≥ 0 for all n ≥ 1

22
Applied Mathematics II

(d) Up to now, we have accumulated number of information about the


sequence. From the facts we have discovered it is known that the sequence is
monotone decreasing and bounded below.
Therefore the sequence is convergent.

Activity 1. Show by counter example that the inverse of Theorem 1.6 is not
true.

Corollary 1.1(Test for divergence): If {a n }∞n=m is unbounded, then {a n }∞n=m


diverges.

 2n 2 + n 
Activity 2. Show that the sequence   is divergent.
 3n + 1  n =1
Definition: Given any sequence {a n } we have the following.

1. A sequence {a n } is called monotone increasing if a n ≤ a n +1 for every n ≥ 1 .

That is
a1 ≤ a 2 ≤ a 3 ≤ K ≤ a n ≤ K

2. A sequence {a n } is called monotone decreasing if a n ≥ a n +1 for every n ≥ 1 .

That is
a1 ≥ a 2 ≥ a 3 ≥ K ≥ a n ≥ K

3. A sequence {an } is monotonic if it is either monotone increasing or monotone

decreasing.
4. A sequence is strictly increasing if a n < a n +1 for all n ≥ 1 . And a sequence is

strictly decreasing if a n > a n +1 for all n ≥ 1 . A sequence is strictly monotonic if it is

either strictly increasing or strictly decreasing.


1 
Example 5. Show that the sequence   is monotone decreasing.
n 
Solution: We want to show that a n ≥ a n +1 for every n ≥ 1 . Notice that

0 < 1 for all n ∈ Ν

23
Applied Mathematics II

⇒ n < n + 1 for all n ∈ Ν


1 1
⇒ < for all n ∈ Ν
n +1 n
Hence the sequence is monotone decreasing.

Activity 3. Show that the following sequences are monotone decreasing.


 n 
i)  
 n + 1
1 
ii)  n 
2 
Theorem 1.8: Every bounded monotonic sequence is convergent.
Proof: Suppose {a n } is an increasing sequence. Since {a n } is bounded, the set

S = {a n : n ≥ 1} has an upper bound. By Completeness Axiom it has a least upper

bound L . Given ε > 0 , L − ε is not an upper bound for S (since L is the least
upper bound). Therefore
a N > L − ε for some integer N .

But the sequence is increasing so a n ≥ a N for every n > N . Thus if n > N we have

a N > L − ε , and a N < L + ε for n > N

So 0 ≤ L − a n < ε since a n ≤ L . Thus L − a n < ε whenever n > N

So lim a n = L .
n →∞

A similar proof works if {a n } is decreasing.

A property of any bounded increasing or decreasing sequence is that it always


has a (finite) limit. This might not seem very useful if what we want is to
actually compute the limit, but in some cases it may help. For instance,
consider the sequence defined recursively in the following way:
a1 = 6 ,

a n +1 = 6 + a n (n ≥ 1) .

24
Applied Mathematics II

It can be shown that a n is a bounded monotone increasing sequence; hence it

has some limit A:


lim a n = A
n →∞

Now, taking limits on both sides of a n = 6 + a n (here we use that the sequence

has a limit) we get that A = 6 + A , i.e., A 2 − A − 6 = 0 . Solving this second degree

equation we get A = 3 or A = −2. Since the sequence is positive, the limit cannot
be negative, hence it must be A = 3:
Then lim a n = 3 .
n →∞

Activity 4. Show that the following sequences are monotonic and determine if
the sequences are convergent by using Theorem 1.8.

 1
a) 2 + 
 n  n =1

 2
b) 4 − 
 n  n =1

 1
c) n − 
 n  n =1
Example 6. Most of the sequences we encounter in real problems do not have
an explicit formula for each term in the sequence. For example, suppose all we
know about the sequence {a n } is that a1 = 4 and

1
a n +1 = a n for n = 1, 2, 3, K .
2
That is, the first term in the sequence is 4 and then each successive term is
one-half of its predecessor. Thus
a1 = 4 ,
a2 = 2 ,
a3 =1,
1
a4 = ,
2

25
Applied Mathematics II

and so on. Hence {a n } is monotone decreasing. Moreover, every term in the

sequence is positive, so a n ≥ 0 for all n . Thus by the Monotone Sequence

Theorem, {a n } converges. Moreover, note that

1
a n +1 = a n
2
implies that
1
lim a n +1 = lim a n ………………..(1)
n →∞ 2 n →∞
If we let
L = lim a n = lim a n +1 ,
n →∞ n →∞

then equation (1) becomes


1
L= L.
2
Hence L = 0 . That is,

lim a n = 0 .
n →∞

Activity 5. Let {an } ∞

n =1
be the sequence 2 , 2 + 2 , 2 + 2 + 2 ,…., where in

general ,
a n+1 = 2 + a n .

a) Show that {an } ∞

n =1
is bounded increasing sequence.

b) Show that {a }

converges to a number k .
n n =1

c) Show that k=2.


1 2 3 n
Activity 6. Let a n = 2
+ 2 + 2 + .... + 2 for each n ≥ 1 .
n n n n
1
a) Show that a n is a Riemann sum for ∫ xdx
0
for each n ≥ 1 .

b) Find lim a n
n →∞

26
Applied Mathematics II

Key terms
• Bounded sequences
• Monotonic sequences

Self test :

True/False
1. The first few terms of a sequence have no effect on the convergence of the
sequence.
2. An increasing sequence that is bounded from below is convergent.
3. If the sequence converges, then the sequence is bounded and monotonic.
4. If all the terms of a given sequence are finite then the sequence is convergent.
5. A sequence obtained by taking difference of two divergent sequences could be
convergent.

Short answer
1
1. If the sequence {a n }n =1 is defined as a1 = 1 and a n = a n−1 + for all n ≥ 2 , then
2n
the value of a101 = ___________________ .

1
2. Given lim a n = α and a1 = 1 , we define a new sequence bn = + (a n − a n +1 ) ,
n →∞ 2n

then ∑b n =1
n = ____________ .

1
(1+ ) n
n
3. The lim e is ________________.
n →∞

(−2) n n 2
4. The lim is________________.
n →∞ ( n + 3)!

4 + 2 sin n
5. The lim is ________________.
n →∞ n2

Workout
1. Prove that the following sequences converge.

27
Applied Mathematics II

2 • 4 • ... • (2n) 1 • 3 • ... • (2n − 1)


a) sn = b) sn =
3 • 5 • ... • (2n + 1) 2 • 4 • ... • (2n)
n

∫e
− x2
2. Let sn = dx . Prove that {sn} converges.
0

3. State whether {sn} converges or diverges. If it converges, find its limit.


1
2n 2 n2 + 2 3n − 1
a) sn = − b) sn = 2 n c) sn = d) sn = n +1 − n
2n − 1 n −1 3n − 3n −1
n! an
4. Prove that i) lim =0 ii) lim = 0 for every real number a
n→∞ nn n → ∞ n!

5. Let α be any real number. Determine the convergence or divergence of the

  α   n 
sequence 1 +    .
  n   

1 1 1
6. Let {a n } be a sequence defined by an = + +K + .
n +1 n + 2 2n
a) Find a 4 − a5 .

b) Show that {a n } converges.



 3n − 2 
7. Prove using ε − N definition that the sequence   converges to the
 4n + 3  n =0
3
number .
4

 π
8. Determine whether the sequence n sin  converges or diverges.
 n  n =1

Summary
• A sequence {a n } is said to have the limit L , written as

lim an = L ,
n →∞

if for any ε > 0 , there is a natural number N such that for every n > N , we
have
an − L < ε .

28
Applied Mathematics II

• If {a n } has a limit, then the limit is unique.

• Given three sequences {a n }, {bn }, {c n } such that

i) a n ≤ bn ≤ c n for every n and

ii) lim a n = L = lim c n ,


n →∞ n →∞

then lim bn = L
n →∞

• {a n } tends to + ∞ if for each positive number k , there is a natural number N

such that
a n > k for all n > N

For such sequences, we write a n → +∞ as n → ∞ or

lim a n = +∞ .
n →∞

• Consider a sequence {a n } .

1
i) If a n > 0 for all n and lim = 0 , then
n →∞ a
n

lim a n = +∞
n →∞

1
ii) If lim a n = ±∞ , then lim = 0.
n →∞ n →∞ a
n

• Let {a n }∞n = m be a sequence, L a number and f a function defined on [m, ∞ )

such that f (n) = a n for all n ≥ m . If lim f ( x) = L , then {a n }∞n = m converges and
x →∞

lim an = L . If lim f ( x) = +∞ (or lim f ( x) = −∞ ) , then {a n }∞n=m diverges, and


n →∞ x →∞ x →∞

lim a n = +∞ (or lim a n = −∞ ). Thus


n →∞ n →∞

lim a n = lim f ( x)
n →∞ x →∞

• Suppose that the sequences {a n } and {bn } converge to limits L and M ,

respectively, and C is a constant. Then


a) lim C = C
n →∞

29
Applied Mathematics II

b) lim Ca n = C lim a n = L
n →∞ n →∞

c) lim (a n + bn ) = lim a n + lim bn = L + M


n →∞ n →∞ n →∞

d) lim (a n − bn ) = lim a n − lim bn = L − M


n →∞ n →∞ n →∞

e) lim (a n bn ) = lim a n lim bn = LM


n →∞ n →∞ n →∞

 a  lim a n L
f) lim  n  = n→∞ = ( if M ≠ 0)
n →∞ b lim
 n  n →∞ n b M


• Let {a n }n = m be a sequence.

i) The sequence {a n }∞n = m is bounded above if there is a number M 1 such

that a n ≤ M 1 for every positive integer n ≥ m .

ii) It is bounded below if there is a number M 2 such that M 2 ≤ a n for

every positive integer n ≥ m .


It is bounded if there is a number M such that a n ≤ M for every n ≥ m .

Otherwise we say the sequence is unbounded.


∞ ∞
• If {a n }n = m converges, then {a n }n = m is bounded.
∞ ∞
• If {a n }n = m is unbounded, then {a n }n = m diverges.

• A sequence {a n } is called monotone increasing if a n ≤ a n +1 for every n ≥ 1 .

That is
a1 ≤ a 2 ≤ a 3 ≤ K ≤ a n ≤ K

• A sequence {a n } is called monotone decreasing if a n ≥ a n +1 for every n ≥ 1 .

That is
a1 ≥ a 2 ≥ a 3 ≥ K ≥ a n ≥ K

• A sequence {an } is monotonic if it is either monotone increasing or

monotone decreasing.
Every bounded monotonic sequence is convergent.

30
Applied Mathematics II

Review Exercises
ln n
1. Show that lim = 0.
n →∞ n

n
2. Show that lim =0.
n →∞ en
 5n 3 − n 2 + 7 n + 2 
3. Find the limit of the sequence  3 2
 , or explain why it does not

 3n + n − n + 10 
converge.
 3n 2 + 2n − 7 
4. Find the limit of the sequence  2
 , or explain why it does not

 n 
converge.
5. In each of the following, determine if the sequence converges of diverges, find
the limit.

 n 
a)  
 n + 2  n =1

 3n 
b)  3 
 n  n +1

n
c)  n 
 e  n =1

d) {[1 + ( c n)] } n ∞
n =1 , where c is any real number.

1 
e)  2 
 n  n =1

 n−3 
f)  
 2n + 4  N =1

3 8 
g)  − n 
 n 5  n =1
6. Evaluate the following limits
8 n + (ln n)10 + n !
a) lim
n →∞ n6 − n!

31
Applied Mathematics II

n
 3n − 1 
b) lim  
n →∞ 4 n + 1 

3n
 1 
c) lim 1 − 
n →∞
 2n + 1 
 2n 
7. Determine whether the sequence   is increasing or decreasing or not
 (3n + 2) 
monotonic.

 sin(kn) 
8. Determine the convergence or divergence of the sequence   where
 n r  n =1
k is a real number and r > 0 .
9. Find sequences {a n } and {bn } such that {a n } and {bn } both diverge, but

{a n + bn } converges.
10. Find sequences {a n } and {bn } such that {a n } diverges, {bn } converges, and

{a n bn } converges.
1 1
11. Show that if < x < , then
n +1 n
n n +1
 1   1
 < (1 + x ) < 1 + 
1/ x
1 +
 n + 1  n
1
Deduce that lim (1 + x ) x exists.
n →∞

1
12. A sequence {a n } is defined by a1 = 1 and a n +1 = for n ≥ 1 . Assume that
(1 + a n )
{a n } is convergent, find its limit.
13. Determine if the following sequences are monotonic and/or bounded.

2
{ }
a) − n 2

n =0 b) {(− 1) }n +1 ∞
n =1 c)  2 
 n  n =5
14. Prove that if lim a 2 n = lim a 2 n +1 , then lim a n exists and
n →∞ n →∞ n →∞

lim a n = lim a 2 n = lim a 2 n +1 .


n →∞ n →∞ n →∞

15. If the sequence {a n } diverges, the sequence {bn } converges, and

32
Applied Mathematics II

lim bn ≠ 0 ,
n →∞

then the sequence {a n bn } diverges.

16. If the sequence {a n } diverges, the sequence {bn } converges, and bn ≠ 0 for all

a 
n then the sequence  n  diverges.
 bn 
17. In each of the following, for an arbitrary ε > 0 , find the smallest integer N

for which a n − L < ε whenever n > N . Verify that your value for N works in

the particular case ε = 0.001 .


1 1
a) an =1 − , L =1 b) an = , L =0
n n2
1 3n3 −1
c) an = 2 , L = 0 d) an = 3 , L = 3
n n
18. Show that a polygon with n equal sides inscribed in a circle of radius r, has
π
parameter Pn = 2rn sin( ) .
n

 3n 
19. Show that  2n 
is a decreasing sequence.
1 + 3  n =1

1 ⋅ 2 ⋅ 3 ⋅ ⋅ ⋅ ( 2n − 10) 
20. Show that   is an increasing sequence.
 n!  n =1
21. Determine the following limits by first showing that each limit is a definite
b
integral ∫ f ( x) dx
a
for a sutable interval [ a, b] and function f .

2
n
i 1 n
n
a) lim ∑   b) lim ∑ 2 2
i =1  n  n i =1 n + i
n →∞ n →∞

Unit Assesment:

• Assignment for submission


• Quiz
• Active participation in group discussion

33
Applied Mathematics II

• Individual and group presentation


• Mid examination
• Final examination
References:
1. Engineering Mathematics, Satsty,S.S(2001),2nd edition.
2. Howard Anton,(1980), Calculus with Analytic Geometry
3. Larson R.E and Hoster R.P,(1979),Calculus with Analytic geometry
4. James Starwart,
5. Calculus and analytic geometry ,Sherman K. Stein , Anthony Barcellos,
Fifth Edition.

34
Applied Mathematics II

CHAPTER 2

REAL SERIES
Introduction: Infinite series are among the most powerful tools in calculus
which are used in analyzing differential equations, in developing methods of
numerical analysis, in defining new functions, in estimating the behavior of
functions, and more.

Objectives: Upon completion of this chapter the student is expected to:


 Test convergence or divergence of any series of constants.
 Differentiate sequence of terms of a series from sequence of partial sums
of a series.
 Use different tests to determine convergence or divergence of series.
 Explain tests of convergence and divergence of a series.
 Classify geometric series which are convergent and those which are
divergent.

Lesson 2.1

Lesson Title: Introduction to series


Introduction: In the previous chapter we have had the techniques to verify
convergence and divergence of sequences. In this section our main focus will be
on the possibility of expressing infinite sums as infinite sequences. Among
infinite sums, which can be expressed as infinite sequences one is the
geometric series. So we will characterize series based on their convergence and
divergence.

Objectives: At the end of this lesson students will be able to:


 Find nth partial sum of a given series.
 Tell similarities between limit of sequence of partial sums of a series and the
sum of the series itself.

35
Applied Mathematics II

 Differentiate convergent geometric series.


 Find sum of convergent geometric series.
In this section, we investigate an important application of infinite series
namely, their use in representing infinite summations.
Suppose the numbers to be added are a1 , a 2 , a 3 , a 4 , L , a k , L . We let

S1 = a1 ,
S 2 = a1 + a 2 ,
S 3 = a1 + a 2 + a 3 ,

M
n
S n = a1 + a 2 + a 3 + L + a n = ∑ a k .
k =1

As n increases without bound, we are led to consider the symbol, or “set of


marks,”

a1 + a 2 + a 3 + L + a n + L , which we shall also denote it by ∑ ak
k =1

The expression
a1 + a 2 + a 3 + L

written alternatively as ∑ ak is called an infinite series. The numbers
k =1

a1 , a 2 , a 3 , a 4 , L , a k , L are called terms of the series.

We can now “add” infinitely many numbers, in certain cases, not by actual
addition, but rather by the process of finding a limit. This is a typical
illustration of the way in which mathematics generalizes familiar concepts to
increase their usefulness.
Example 1. The following are examples of series.
1 1 1
(a) 1 + + + +L
2 3 4
1 1
(b) 1 + 2
+ 2 +L
2 3

36
Applied Mathematics II


Definition: Given a series ∑ a k , its n th partial sum S n is given by
k =1

n
S n = ∑ a k = a1 + a 2 + a 3 + L + a n
k =1


The sequence {S n } is called the sequence of partial sums of the series ∑ ak .
k =1

Example 2. Consider the series 1-1+1-1+1-1+ ⋅ ⋅ ⋅ .


Then S 2 n −1 = 1 and S 2 n = 0 (Explain why?)

Definition: Consider the sequence of partial sums {S n } of the series ∑ ak .If this
k =1


sequence converges to a number S , then we say that the series ∑ ak
k =1

converges to S and write



∑ a k = nlim
→∞
Sn = S
k =1

On the other hand if the sequence of partial sums {S n } diverges, then we say

the series ∑ ak diverges.
k =1

We shall illustrate the method of finding the “sum” of an infinite series for the
case of the repeating decimal
3 3 3
0.3333K = + + +L
10 100 1000
Here
3
S1 =
10
3 3
S2 = +
10 100
M
3 3 3
Sn = + 2 +L+ n
10 10 10

37
Applied Mathematics II

We can obtain an explicit algebraic expression for S n as follows. We can

1
multiply both sides of the equation for S n by and obtain
10
1 3 3 3
S n = 2 + 3 + L + n +1
10 10 10 10
Subtracting this from S n , we get

1 3 3 3 1 
Sn − Sn = − n +1 = 1 − n 
10 10 10 10  10 
Therefore
3 1 
Sn = 1 − n 
9  10 
n
1
Clearly, as n approaches ∞ ,   approaches 0, and
 10 
3 1
lim S n = =
n →∞ 9 3
We can therefore write the sum of the infinite series:
1 3 3 3 1
+ 2 + 3 +L+ n +L=
10 10 10 10 3
The repeating decimal illustration is a special case of a geometric series which
will be discussed on the following example.
Geometric series comprises one of the few classes of series for which we can
evaluate sums exactly. For most series we can only approximate the sum by
computing the partial sums S n for sufficiently large values of n.

Example 3 (Geometric Series). Let a ≠ 0 . Consider the series



∑ ar n = a + ar + ar 2 + ar 3 + L .
n =0

Then, employing the technique used above, the partial sum becomes
 1− rn

2
S n = a + ar + ar + L + ar n −1
(
= a1+ r +L+ r n −1
) = a 1− r r ≠ 1 why?
a ( n + 1) r =1

38
Applied Mathematics II

a
when − 1 < r < 1 , S n → as n → ∞ . Why?
1− r
When r > 1 , S n diverges because r n → +∞ as n → ∞ . Why?

When r = 1 , S n = a(n + 1) diverges. Why?

When r = −1 , S n =
[
a 1 − ( −1) n ]
diverges. Why?
2
When r < −1 , S n diverges because r n → ±∞ . Why?

Therefore, the geometric series ∑ ar n converges if and only if − 1 < r < 1 , and,
n =0


a
∑ ar n = 1 − r for − 1 < r < 1 .
n =0


1
Example 4. Show that ∑( 2)
n =1
n −1
converges and find its sum.

1
Solution: Using Example 3 with a = 1 and r = , that
2
∞ n −1
1 1 1 1 1 1
∑  2  =1+ + + +
2 4 8 16
+L=
1
= 2.
n =1 1−
2
Activity 1. Find the sum of the following infinite series whenever possible.
∞ n −1
1
a) ∑  
n =1  3 


b) ∑ 4(0.21) n−1
n =1


2
c) ∑ 7n
n =1

Key terms
• Series
• Sequence of partial sums
• Geometric series
Self Test.

39
Applied Mathematics II

1. Compute the fourth partial sum of each series


∞ ∞
1 ∞
(−1) n
a) ∑ (−1) n
b) ∑ ( ) n c) ∑
n =1 n =1 5 n=2 n
2. Find a formula for the partial sums of the seies . For each series, determine
the partial sums have a limit. If so, find the sum of the series.

1 ∞
1 ∞
4n + 2 ∞
n +1 − n
a) ∑ ( )n
n =1 3
b) ∑ 2
n =1 (9n + 3n − 2)
c) ∑
n =1 5
n −1
d) ∑
n =1 n2 + n

1
3. Show that ∑ ln(1 − n
n =1
2
) = − ln 2

1 1 1 1
4. Show that + + + ... =
1 .3 3 .5 5 .7 2
6n

5. Show that ∑ n +1 =2
n =1 (3 − 2n +1 )(3n − 2n )

Lesson 2.2 and 2.3

Lesson Title: Properties of Series


Introduction: In this section we will discuss algebra of sequences and the
effect of adding or subtracting finite terms from a sequence on the convergence
or divergence of the original sequence. In addition to the properties of
sequences we will talk about the Divergence Test which is the most common
test to find out divergence of a series. We will also discuss some of the special
types of series, like the harmonic series and telescoping series.

Objectives: Upon the accomplishment of this lesson the student will be able to:
 Apply the Divergence test for series whenever necessary.
 Use algebra of series to simplify the problem of finding convergence or
divergence of series.
 Explain why the harmonic series is divergent.
 Find sum of telescoping series.

40
Applied Mathematics II

 Tell the effect of adding or subtracting finite terms from a series on the
convergence or divergence of the series.

Theorem 2.1: Suppose that the sequences {a n } and {bn } for the terms of
∞ ∞
convergent infinite series ∑ a n = A and ∑ bn = B .
n =1 n =1

Then the following series are all convergent to the values given:
∞ ∞ ∞
1. ∑ (a k + bk ) = ∑ a k + ∑ bk = A + B
k =1 k =1 k =1

∞ ∞ ∞
2. ∑ (a k − bk ) = ∑ a k − ∑ bk = A − B
k =1 k =1 k =1

∞ ∞
3. ∑ ca k = c∑ a k = cA if c is any real constant.
k =1 k =1

Activity 1. Prove Theorem 2.1.


 1  n  1  n  ∞  1  n ∞  1  n

Example 1. ∑   +    = ∑   + ∑  
 3 
n =1   4   n =1  3  n =1  4 

1  1  2  1  1 2 
=  +   + L +  +   + L
 3  3    4  4  

1  1 1  1  1  1 2 
2
= 1 + +   + L + 1 + +   + L
3  3  3   4  4  4  
1 1 1 1
= +
3 1− 1 4 1− 1
3 4
5
= .
6
∞  3n − 2 n 
Activity 2. Find the sum of the series ∑  6 n .

n =0  
Activity 3. If ∑ an is convergent and ∑ bn is divergent, show that the series

∑ (a n + bn ) is divergent.

41
Applied Mathematics II

Comparison of series with different initial indices


∞ ∞
Let us compare the partial sums of two series ∑ a n and ∑ a n . Notice that for
n=2 n =6

any positive integer k ≥ 5 .


1 1 1 1 1 1 1 1 1 1 1
3
+ 3 + 3 +L + 3 =  3 + 3 + 3 + 3  +  3 + 3 + L + 3 
2 43442
1 4 4443 k  2444
1 3 24 3 1644
4 454 74244k4 3
∞ 1 ∞
k th partial sum of ∑ k th partial sum of ∑
1
n3
n=2 3
n = 6n

∞ ∞
1 1
From which it follows that if ∑ n3 converges, then so does ∑ n3 , and vice
n=2 n =6

versa. Moreover, if the two series converge, their sums are related by

 1
1 1 1 1  ∞ 1
∑ 3  2 3 33 4 3 5 3  + ∑ n 3
=  + + +
n=2 n n =6

A similar argument would show that if the second series is convergent.


∞ ∞
In general, the series ∑ a n converges if and only if the series ∑ an converges.
n = m1 n = m2

In addition, if the two series converge, then


∞ ∞
(
∑ an = am + am 1 1 +1 + a m + 2 + L+ a m
1 2 −1
)+ ∑ a n , for m 2 > m1
n = m1 n = m2


Thus the index m at which the summation of the series ∑ an begins is
n=m

irrelevant to the convergence of the series although the actual sum of the series
is affected by the index m . For that reason, although most of our theorems will

be stated for series of the form ∑ an (with initial index 1), all these theorems
n =1


can be applied to a series ∑ an with arbitrary initial index m . We summarize
n=m

the above result in the following theorem.


Theorem 2.2: The convergence or divergence of a series is unaffected if a finite
number of terms are inserted, deleted or altered.

42
Applied Mathematics II

Activity 4. Prove Theorem 2.1.



Theorem 2.3: If ∑ ak converges, then lim a k = 0 .
k →∞
k =1

Proof: Notice that,


S n = a1 + a 2 + L + a n and , S n −1 = a1 + a 2 + L + a n −1

Then, a n = S n − S n −1 .

Since ∑ ak converges by assumption, we know that lim S n exists and also
n →∞
k =1

lim S n −1 exists, with


n →∞

lim S n = lim S n −1 = S , where S is a number.


n →∞ n →∞

Now, we have lim a n = lim (S n − S n−1 ) = lim S n + lim S n −1 = S − S = 0 , which complete


n →∞ n →∞ n →∞ n →∞

the proof.

Corollary 2.1(Divergence test): If lim a k ≠ 0 (or does not exist), then
k →∞
∑ ak
k =1

diverges.
Proof: It is the contra-positive of Theorem 2.1.

Example 2. The series ∑ (− 1)n is divergent because the limit of the n th term
n =1

(− 1)n does not exist.



n!
Example 3. The series ∑ n2 is divergent because
n =1

n! 1
lim 2
= = +∞ ≠ 0 .
n →∞ n n2
lim
n →∞ n !


n2
Activity 5. Show that ∑ 2
+4
is divergent.
n = 0 5n

43
Applied Mathematics II

Activity 6. A ball is dropped from a height of 4ft. Each time it strikes the
3
ground after falling from a height of h ft, it rebounds a distance h ft. Find the
4
total distance traveled by the ball.
Remark. The divergence test is a “one way” test, because lim a k = 0 does not
k →∞


imply that the series ∑ ak converges. As we will see in the following example,
k =1

there are series for which the terms approach 0, but the series still diverges.
Example 4 (Harmonic Series). Consider the harmonic series

1 1 1 1 1
∑ n = 1 + 2 + 3 + 4 + L . For this series lim a n = lim
n →∞ n →∞ n
= 0 . But we can show that
n =1


1
∑n diverges.
n =1

Fig2.1

Now we can apply corollary 1.1 to show divergence. i.e. we can demonstrate
that there is no constant number M that is greater than or equal to every
partial sum.
Notice that the sequence of partial sums

44
Applied Mathematics II

1 1 1 1 1 1
S1 = 1 , S 2 = 1 + , S3 = 1 + + , S = 1 + + + , L form an increasing
2 2 3 2 3 4
sequence. To this end we will consider some selected partial sums, namely
S 2 , S 4 , S 8 , S16 , K .Note that the subscripts are successive powers of 2, so that

there are the partial sums of the form S n .These partial sums satisfy the
2

inequalities;
1
S2 =1 +
2
1 1 1 1 1 1 1 1 1
S4 =1 + + +  > 1 + +  +  = 1 + + = 1 + 2 
2 3 4 2 4 4 2 2 2

1 1 1 1 1 1 1 1  1 1 1 1 1 1
S8 = 1 + + +  +  + + +  >1+ +  +  +  + + + 
2 3 4 5 6 7 8 2  4 4 8 8 8 8
1 1 1 1
= 1 + + + = 1 + 3 
2 2 2 2

1 1 1 1 1 1 1 1 1 1 1 1 1 1 1
S16 = 1 + + + + + + + + + + + + + + + 
2  3 4   5 6 7 8   9 10 11 12 13 14 15 16 
1 1 1 1 1 1 1  1 1 1 1 1 1 1 1
>1+ +  +  +  + + +  +  + + + + + + + 
2  4 4   8 8 8 8   16 16 16 16 16 16 16 16 
1 1 1 1
=1+ + + = 1 + 3 
2 2 2 2
5 6 n
Similarly S 32 > 1 + , S 64 = 1 + , and in general S n > 1 + for n > 1 .
2 2 2 2
Now, if M is any constant, we can certainly find a positive integer n such that
n+2 n
> M . But for this n ; S n > 1 + > M .
2 2 2
So that no constant M is greater than or equal to every partial sum of the
harmonic series, which proves divergence of the series.
It’s now time to look at another special type of series. In the next example we
are going to look at a series that is called a telescoping series. The name in this
case comes from what happens with the partial sums.

45
Applied Mathematics II


1
Example 5 (Telescoping Series). Show that the series ∑ n(n + 1) is convergent
n =1

and find its sum.


Solution: To show convergence of this series we can compute the partial sums.
n
1 1 1 1 1
Sn = ∑ = + + +L+
k =1 k ( k + 1) 1⋅ 2 2 ⋅ 3 3 ⋅ 4 n(n + 1)
We can simplify this expression if we use the partial fraction decomposition
1 1 1
= −
k (k + 1) k k + 1
Hence we have
n n
1 1 1 
Sn = ∑ = ∑ − 
k =1 k ( k + 1) k =1  k k + 1

 1  1 1 1 1  1 1 1 1 
= 1 − + − + −  +L+  − + − 
 2   2 3  3 4  n −1 n   n n + 1

 1 1  1 1  1 1  1 1 1
=1+ − +  + − +  + − +  + L + − +  −
 2 2  3 3  4 4  n n n +1
This implies
1
Sn = 1 − .
n +1
 1 
So lim S n = lim 1 −  =1
n→∞ n → ∞ n +1
Therefore the given series is convergent and


1
∑ n(n + 1) = 1 .
n =1


4
Example 6. Consider ∑ (4n − 3) (4n + 1) . We have
n =1

4
an =
(4n − 3) (4n + 1)

46
Applied Mathematics II

1 1
= −
4n − 3 4n + 1

1 1
= −
4n − 3 4(n + 1) − 3
Thus we have,
n
4 n
 1 1 
Sn = ∑ =∑ − 
k =1 ( 4k − 3) ( 4k + 1) k =1  4k − 3 4( k + 1) − 3 

 1 1 1 1 1   1 1   1 1 
= 1 −  +  −  +  −  + L +  −  +  − 
 5   5 8   8 11   4(n − 1) − 3 4n − 3   4n − 3 4(n + 1) − 3 

 1 
= 1 − 
 4( n + 1) − 3 
Therefore the given series is convergent and

4
∑ (4n − 3) (4n + 1) = 1.
n =1


1
Activity 7. Find the sum of the series ∑ n (n + 1)(n + 2) .
n =1


n 2 + 3n + 1
Activity 8. Show that the series ∑ n 2 (n + 1) 2 is convergent.
n =3

Activity 9. Let {a n }∞n=1 be a sequence. Show that the n th partial sum of the

series ∑ (a n −a n+1 ) is a1 − a n . Conclude that this series converges if and only if
n =1

lim a n exists, in which case the sum is a1 − lim a n .


n →∞ n→∞

Key terms
• Divergence test
• Harmonic series
• Telescoping series
Self Test.

47
Applied Mathematics II

1. Use the divergence test to show that the series diverges.



n+2 ∞ ∞
en
a) ∑ b) ∑ arctan n c) ∑
n =1 3n + 4 n =1 n =1 n

2. Find the sum of the following series.



7 6  ∞
 7 6 
a) ∑  n +  b) ∑  2 − n −1  .
 n − 1 10 
n =1  3 (n + 3)(n + 4)  n =1

Lesson 2.4

Lesson Title: Tests for convergence of a series of constants.


Introduction: For series discussed in the previous three lessons we were able
to find a simplified expression for the n th partial sum which then simplifies the
problem of determining convergence or divergence of the series given. However,
it is an arduous task or sometimes impossible to find a simplified form for the
n th partial sum for most of the series we encounter. Hence in this section our
aim will be on finding out divergence or convergence without calculating the
sum by using the Integral Test.

Objectives: After a successful accomplishment of this lesson the student will


be able to:
 Apply the Integral Test to test divergence or convergence of positive series.
 State the conditions to apply Integral Test to test convergence or divergence
of a series.
 Identify a p-series.
 Differentiate convergent p-series from divergent p-series.

Tests for convergence of a series of constants


In general it is difficult to find the exact sum of a series. We were able to
accomplish this for geometric series and the telescopic series because in those
cases there was a simple formula for the n th partial sum S n . But usually it is

not easy to compute S n . This is true when we can’t find any simpler expression

48
Applied Mathematics II

for the sum of the first n terms of a series as a function of n . Therefore in the
next few sections we shall develop several tests that will enable us to say
whether a series is convergent or divergent without explicitly finding its sum.
These are tests that depend on the individual terms of the series rather than on
their sums.
Consider a series of numbers

∑ a k = a1 + a 2 + a3 + L + a n + L
k =1

By definition of convergence, we say that the series converges if the sequence


S n of partial sums

S1 = a1 ,
S 2 = a1 + a 2 ,
S 3 = a1 + a 2 + a 3 ,

M
n
S n = a1 + a 2 + a 3 + L + a n = ∑ a k
k =1

tend to a definite finite limit as n increases without bound. If the partial sums
S n do not have such a limit, then the series is said to diverge.

Non-Negative series

Suppose that all the terms a k of a series ∑ ak are non-negative. Then when we
k =1

calculate the partial sums S1 , S 2 , S 3 and so on, we see that each one is greater

than or equal to its predecessor, since S n +1 = S n + a n +1 . That is,

S1 ≤ S 2 ≤ S 3 ≤ L ≤ S n ≤ S n+1

Therefore the sequence {S n } is monotone increasing. Thus, to show convergence

of this series it suffices to verify that the series is bounded. (Why?)

49
Applied Mathematics II

Integral Test
We shall start this section by recalling some facts about improper integral.
Improper integrals are integrals with infinite intervals of integration or integrals
in which the integrand becomes infinite within the interval of integration. If f
is continuous on the interval [a, + ∞] , then we define the improper integral of
∞ ∞ b

∫ f ( x)dx as the limit: ∫ f ( x)dx = blim


→∞ ∫
f ( x)dx .
a a a

Theorem 2.4: Suppose f is a continuous, positive, decreasing function on



[1, ∞) and let a n = f (n) . Then the series ∑ an is convergent if and only if the
n =1


improper integral ∫ f ( x)dx is convergent. In other words:
1

∞ ∞
a) If ∫ f ( x)dx is convergent, then ∑ an is convergent.
1 n =1

∞ ∞
b) If ∫ f ( x)dx is divergent, then ∑ an is divergent.
1 n =1

Proof: The basic idea behind the Integral Test can be seen by looking at fig.2.2.
and fig2.3. The area of the first shaded rectangle in fig2.1 is the value of f at

the right endpoint of [1 , 2] , that is, f (2) = a 2 . So comparing the area of the
shaded rectangle with the area under y ( x) = f ( x) from 1 to n , we see that
n

∫ f ( x)dx ≤ a1 + a 2 + a3 + L + a n−1 ………………..(1)


1

Likewise fig2.3 shows that


n
a 2 + a 3 + L + a n ≤ ∫ f ( x)dx ……………………….(2)
1

50
Applied Mathematics II

Fig2.2

Fig2.3


If ∫ f ( x)dx is convergent, then (2) gives
1

n n ∞

∑ a k ≤ ∫ f ( x)dx ≤ ∫ f ( x)dx
k =2 1 1

Since f ( x) ≥ 0 . Therefore
n ∞
S n = a1 + ∑ a k ≤ a1 + ∫ f ( x)dx = M , let us say.
k =2 1

51
Applied Mathematics II

Since S n ≤ M for all n , the sequence {S n } is bounded above.

Also S n +1 = S n + a n +1 ≥ S n , since a n +1 = f (n + 1) ≥ 0 .

Thus {S n } is a bounded monotone increasing sequence and so it is convergent



by Theorem1.7. This means that ∑ an is convergent.
n =1

∞ ∞
If ∫ f ( x)dx is divergent, then ∫ f ( x)dx → ∞ as n → ∞ because f ( x) ≥ 0 . But
1 1

inequality (1) gives


n n −1

∫ f ( x)dx ≤ ∑ a k = S n −1
1 k =1


And so S n −1 → ∞ . This implies that S n → ∞ and so ∑ an diverges. (Why?)
n =1

Note: When using Integral Test it is not necessary to start the series or the
integral at n=1. For instance, in testing the series
∞ ∞
1 1
∑ n(ln n) we can use ∫ x(ln x )dx
n=2 2


1
Example 1( p-Series). Show that the series ∑ np converges if and only if p > 1 .
n =1

1
Solution: If p ≤ 0 , then does not tend to 0 as n → ∞ and so, by divergence
np

1
test ∑ np diverges.
n =1

1
Assume that p > 0 . Let f ( x) = on [1 , ∞) . Then f (x) is continuous, positive
xp
1
and monotone decreasing and f (n) = .
np
Now
∞ ∞ b

∫ f ( x)dx = ∫ x dx = lim ∫ x − p dx
−p
b →∞
1 1 1

52
Applied Mathematics II

  − p +1 b 
 lim  x  , p ≠1

= b→∞ − p + 1 1


 b
 blim ln x 1 , p =1
→∞

For p > 1 , − p + 1 < 0 and b − p +1 → 0 as b → ∞ , so the integral and the series


converge.
For 0 < p < 1 , − p + 1 > 0 and b − p +1 → ∞ as b → ∞ , so the integral and the series
diverge.
For p = 1 , ln b → +∞ as b → ∞ , so the integral and the series diverge.

1
∴ The series ∑ np converges if and only if p > 1 .
n =1


1
Example 2. Test the series ∑ k2 +1 for convergence or divergence.
k =1

Solution: If we replace k by x in the formula for a k , we obtain the function

1
f ( x) = 2
which is continuous, positive and decreasing on [1 , ∞) . Why?
x +1
So we use the Integral Test:
∞ b
1 1 b
∫ 2
x +1
dx = lim ∫ 2
b →∞ x + 1
dx = lim arctan
b →∞ 1
1 1

 π
= lim  arctan b − 
b →∞ 4
π π π
= − = which show that the improper integral converges and
2 4 4
hence the given series converges.

Example 3. Determine if the following series is convergent or divergent.



1
∑ n ln n
n=2

Solution: In this case the function we will use is,

53
Applied Mathematics II

1
f ( x) =
x ln x
This function is clearly positive and if we make x larger the denominator will
get larger and so the function is decreasing. Therefore, all we need to do is
determine the convergence of the following integral.
∞ b
1 1
∫ x ln x
dx = lim ∫
b →∞ x ln x
2 2

b
= lim (ln(ln x) ) 2
b →∞

= lim (ln(ln b) − ln 2) = ∞
b →∞

The integral is divergent and so the series is also divergent by the Integral Test.

1
Example 4. The series ∑ n (ln n)
n=2
k
converges if and only if k > 1 .

1
Solution: Let f ( x) = on [ 2 , ∞) . Then f (x) is continuous, positive. We will
x(ln x) k
check that f (x) is eventually monotone decreasing.
From
1
f ' ( x) = [ x −1 (ln x) − k ] ' = x − 2 (ln x) − k − k x −1 (ln x) − k −1
x
= − x −2 (ln x) − k −1 (ln x + k )
we have f ' ( x) ≤ 0 when ln x > − k . Thus f (x) is monotone decreasing when
ln x > − k and so f (x) is eventually monotone decreasing.
Now
∞ ∞
1
∫ f ( x)dx = ∫ x(ln x) k dx
2 2

1
Let y = ln x , then dy = dx . Substituting this in the above integral we get;
x
∞  1 ∞
1  y − k +1 , k ≠1
∫ k
dy =  − k + 1 ln 2
ln 2 y ln(∞) − ln(ln 2) , k =1

54
Applied Mathematics II

∞ ∞
1
Thus ∫ f ( x)dx converges if and only if k > 1 and so the series ∑ n(ln n) k
2 n=2

converges if and only if k > 1 .


ln n
Example 5. Determine whether the series ∑ converges or diverges.
n =3 n

ln x
Solution: The function f ( x) = is positive and continuous for x > 1 since the
x
logarithm function is continuous. But it is not obvious that whether or not f
is decreasing, so we compute its derivative!
1
x − ln x
x 1 − ln x
f ' ( x) = 2
=
x x2
Thus f ' ( x) < 0 when ln x > 1 , that is , x > e . It follows that f is decreasing when
x > e and so we can apply the Integral Test.
∞ b
ln x ln x
∫ x dx = blim
→∞ ∫ x
dx
3 3

(ln x) 2 (ln b) 2 (ln 3) 2
= lim = lim −
b →∞ 2 3
b →∞ 2 2

ln n
Since this improper integral is divergent, the series ∑ is also divergent by
n =3 n

the Integral Test.


Example 6. Determine if the following series is convergent or divergent.
∞ 2
∑ n e −n
n=2

Solution: In this case the function we will use is,


2
f ( x) = x e − x
This function is always positive on the interval that we are looking at. Now we
need to check that the function is decreasing. It is not clear that this function

55
Applied Mathematics II

will always be decreasing on the interval given. To verify this we can use our
Applied Mathematics I knowledge. So, the derivative of this function is
2
f ' ( x) = e − x (1 − 2 x 2 )

This function has two critical points (which tell us where the derivative changes
1
sign) at x = ± . Since we are starting at n = 0 we can ignore the negative
2
critical point. Picking out a couple of test points we can see that the function is
 1   1 
increasing on the interval 0 ,  and it is decreasing on  , ∞  . Therefore,
 2  2 
eventually the function will be decreasing and that’s required for us to use the
Integral Test. So,

∞ b
− x2 2
∫ xe dx = lim ∫ x e − x dx
b→∞
0 0

b
 1 2 
= lim  − e − x 
b →∞  2  0

1 1 2  1
= lim  − e −b  =
b →∞  2 2  2

This integral is convergent and so the series must be convergent by the Integral
Test.
Warning!
The integral test has two major problems.
1. The function used in writing down the series might not be functions of a
real number x . What does this mean? Suppose that we are trying to
1 1
study the infinite series ∑ n! . We can’t do this by setting f ( x) =
x!
,

because x ! makes no sense if x isn’t an integer. This problem usually

56
Applied Mathematics II

occurs in series with factorials in them, and in that case usually ratio test
is the way to decide about convergence or divergence.
2. The integral that we get might be hard or even impossible to handle.
1
Suppose that we are looking at ∑ . We’d like to decide if this
n3 + 1

dx
converges by looking at ∫ x3 + 1
1

But that integral doesn’t show up in the tables.


We will see how to handle such type of problems when we talk about the
limit comparison test.

Activity 1. Use the integral test to see if these series converge or diverge.

1
a) ∑ n2 + 1
n =1


1
b) ∑ n2 − 2
n =1


1
c) ∑
n =1 n2 + 2

Theorem 2.5: For a positive series ∑ a k , the sequence of partial sums {S n } is
k =1

monotone increasing.
Activity 2. Prove Theorem 2.3.
Corollary 2.2: a) If {S n } is bounded for a positive series, then the series

converges.
b) If {S n } is not bounded from above, then the series diverges.

Activity 3. Prove corollary 2.2.


Key terms
• Integral test
• P-series
Self Test

57
Applied Mathematics II


1
1. ………….Prove that ∑ n(ln n)
n=2
p
converges for p > 1 and diverges for p ≤ 1 .


1
2. Determine those value of p for which the series ∑ (n + 2)
n =1
p 2 − p +1

converges.
3. Check the convergence or the divergence of the following series.
∞ ∞ ∞
arctan n 1
a) ∑ ∑ n 2e n ∑ 9n
−3

2
b) c) 2
n =1 n + 1 n =1 n =1 +1

Lesson 2.5 & 2.6

Lesson title: Comparison Test


Introduction: In this section we continue our discussion of the convergence
properties of infinite series. So far we have two classes of series, namely, the
geometric series and the p-series, for which classification as either convergent
or divergent is relatively easy. In this section we will consider a method for
determining the convergence or divergence of a series by comparing a given
series with a series which is already known to converge or diverge. We can use
the geometric series and the p-series to verify convergence or divergence of a
given series In order to make significant use of such a result it is necessary to
have a supply of series whose convergence or divergence is already known.

Objectives: After a successful completion of this lesson the student will be able
to:
 Use comparison test to determine convergence or divergence of a given
series whenever possible.
 Explain the conditions where comparison test can be applied to find out
convergence or divergence of a certain series.
 Apply the limit comparison test to determine convergence or divergence of
series.

58
Applied Mathematics II

 Recall the geometric series and p-series whose convergence or divergence


are known which can be used for comparison purpose..

∞ ∞
Theorem 2.6(Comparison Test): Consider two positive series ∑ a k and ∑ bk .
k =1 k =1

Suppose that eventually 0 < ak ≤ bk .


∞ ∞
i) If ∑ bk converges, then ∑ ak converges.
k =1 k =1

∞ ∞
ii) If ∑a
k =1
k diverges, then ∑b
k =1
k diverges.

n n ∞
Proof: i) Let An = ∑ a k , Bn = ∑ bk and B = ∑ bk . Since both series have positive
k =1 k =1 k =1

terms, the sequences {An } and {Bn } are monotone increasing. Also Bn → B , so

Bn ≤ B for all n . Since a i ≤ bi , we have An ≤ Bn for all n . Thus An ≤ B for all n .

This means that {An } is monotone increasing and bounded above and therefore

converges by corollary 2.2 above. Thus the series ∑ ak converges.
k =1


ii) If ∑a
k =1
k is divergent, then An → ∞ (since {An } is increasing). But bi ≥ ai so


Bn ≥ An . Thus Bn → ∞ . Therefore ∑b
k =1
k diverges.

Example 1. The series



1
∑ n2 + 1
n =1

converges since
1 1
0< 2
< 2 for all n
n +1 n

1
and ∑ n2 is convergent series (namely, a p-series with p=2).
n =1

Example 2. The series

59
Applied Mathematics II


1
∑ 2n − 1
n =1

Diverges since
1 1
> > 0 for all n.
2n − 1 2n

1
and ∑ 2n is a divergent series ( since it is a constant multiple of the
n =1

harmonic series).
∞ k
 2k − 1 
Example 3. The series ∑   converges because
k =1  3k + 2 

k k
 2k − 1  2
  ≤ 
 3k + 2   3
∞ k
2
and the geometric series ∑   is convergent.
k =1  3 

Activity 1. Use Comparison Test to show convergence or divergence of the


following series.

1
a) ∑ n3 + n + 7
n =1


(sin n )2
b) ∑
n =17 n(n + 1)


n+3
c) ∑n n+2
n =1


1
Example 4. Determine convergence or divergence of the series ∑ 3k + 2 .
k =1


1
Solution: We know that the geometric series ∑ 3k converges. Since
k =1


1 1 1
0≤ k
3 +2

3 k
for k ≥ 1 it follows from comparison test that ∑ 3k + 2 converges.
k =1


1 1 1
Example 5. The series ∑3 k diverges because
3
k

k
k =1

60
Applied Mathematics II


1
and the harmonic series ∑k diverges.
k =1


Remark. 1. Suppose 0 < ak ≤ bk and ∑b
k =1
k diverges. Then no conclusion can be


drawn about the divergence or convergence of ∑a
k =1
k .


2. Similarly, suppose 0 < ak ≤ bk and ∑ ak converges. Then no
k =1

conclusion can be drawn.



1
Example 6. Show that ∑ 3k − 2 converges.
k =2

1 1
Solution: Notice that k
≤ k
for k ≥ 2 .
3 3 −2
Thus it is impossible to determine the convergence or divergence of the given

1
series by it with the series ∑ 3k
k =1

But
1 1 1
≤ =
k
3 −2 k
3 −3 k −1
( ) for k ≥ 2 .
2 3 k −1

1 1 ∞  1 
and since ∑ 2(3 k −1 ) = ∑   is convergent, the comparison Test implies that
2 k =1  3 k −1 
k =2

the given series converges as well.



1
Activity 2. Use the comparison Test to show that the series ∑ (n + 1) 2 is
n =1

convergent.
Activity 3. Determine convergence or divergence of the following series.

n
a) ∑ n 2 − cos 2 (n)
n =1


sin 2 (n)
b) ∑ n2
n =1

61
Applied Mathematics II


1
c) ∑ n 3n
n =1

Theorem 2.7(The Limit Comparison Test)


Suppose that ∑ an and ∑ bn are series with positive terms.

an
i) If lim = c > 0 , then either both series converge or both diverge.
n →∞ b
n

an
ii) If lim
n →∞ b
= 0 and ∑ bn converges, then ∑ an also converges.
n

an
iii) If lim
n →∞ b
= ∞ and ∑ bn diverges, then ∑ an also diverges.
n

c
Proof: i) Let ε = in definition of convergence (or divergence ) of sequences. And,
2
an
since lim = c , there is an integer N such that
n →∞ b
n

an c
− c < when n > N
bn 2

c a n 3c
Thus < < when n > N
2 bn 2
And so
c  3c 
 bn < a n <  bn when n > N ……………………(*)
2  2
 3c 
If ∑ bn converges, so does ∑  2 bn , why? The right half of the above inequality

(*) then shows that ∑ an converges by comparison test. It then follows that
n= N

∑ an converges.

c
If ∑ bn diverges, so does ∑  2 bn and the left half of (*) together with the

comparison test shows that ∑ an diverges.

Activity 4. Prove part (ii) and (iii) of Theorem 2.4.

62
Applied Mathematics II


1
Example 7. Show using Limit comparison that the series ∑ 2n − 1 converges.
n =1

1 1
Solution: Let a n = n
, bn =
2 −1 2n
an 2n 1
So lim = lim n = lim = 1.
n →∞ b n→ ∞ 2 − 1 n →∞ 1 − 1 2 n
n


1
Since the limit exists and ∑ 2n is a convergent geometric series, the given
n =1

series converges by the Limit Comparison Test.



1
Example 8. Determine convergence or divergence of ∑ (ln n)k , where k is a
n=2

constant.
1 1
Solution: Let bn = and a n = . Then
(ln n) k
n

bn 1 (ln n) k n
lim = lim = lim =∞,
n →∞ a n→∞ 1n n → ∞ (ln n ) k
n

∞ ∞
1 1
Since the series ∑n is divergent then the series ∑ (ln n)k is divergent for
n=2 n=2

any k .

1
Example 9. Show that ∑ (an 2 + bn + c ) is convergent where a , b and c are
n =1

positive real numbers.



1
Solution: We know that ∑ n2 is convergent.
n =1

1 1
If we let a n = 2
, bn = 2
n an + bn + c

63
Applied Mathematics II

1
an n2 an 2 + bn + c
= =
bn 1 n2
Then
(an 2
+ bn + c )
b c
=a+ + 2
n n

an 1
so that lim
n →∞ b
= a > 0 . Thus by the Limit Comparison Test ∑ (an 2 + bn + c ) is
n n =1

convergent.
Activity 5. Use the Limit Comparison Test to show that the series
n
∞  tan −1 n 
∑  2  is convergent.
n=2  

1
Example 10. Show that the series ∑ 3
is convergent.
n =1 n +1
1
Solution: Let a n = . The most important term in the numerator is the only
n3 + 1

term in the numerator: 1. The most important term in the denominator is n3 .


1
Therefore we try to let bn = , and see if this is good enough. We have check
3
n
what will happen if we use the limit comparison test:

an 1 n3 + 1
lim = lim
n →∞ b n →∞
n 1 n3

n3
= lim
n→∞ n3 + 1

1
= lim =1 .
n→∞ 1
1+
n3

64
Applied Mathematics II

But how do we decide if ∑ bn converges or diverges? We know that this series is

3
a p-series with p =
2
> 1 . Hence the series ∑ bn is convergent. Therefore the


1
series ∑ is convergent.
n =1 n3 + 1

n 2 + 3n + 2
Activity 6. Test convergence or divergence of the series ∑ n3 + 5
.
n =1

Key terms
• Comparison test
• Limit comparison test

Self Test.
1. Use the comparison test or limit comparison test and determine the
convergence or divergence of the following series.
∞ ∞ ∞ ∞
n 1 1 n
a) ∑ 3
n =1 n + 1
b) ∑n
n=2
2
n −1
c) ∑e
n =1
n 2 d) ∑n
n =1
2
−3
∞ ∞

∑ an is a convergent nonnegative series , then so is ∑a


2
2. Prove that if n .
n =1 n =1

Lesson 2.7

Lesson title: The Ratio Test


Introduction: In the previous section we saw that we could demonstrate the
convergence of a positive series by comparing it with another positive series
which is already known to converge. Both of the techniques developed, the
comparison test and the limit comparison test, provide to be very useful;
however, they both suffer from drawback of requiring that we first find a series
of known behavior which allows for the proper comparison with the series
under consideration. In this section we shall consider another test for
convergence, the Ratio Test, which determines whether or not the terms of a
series are approaching 0 at a rate sufficient for the series to converge without

65
Applied Mathematics II

reference to any other series; it has the limitation of being inconclusive in


certain circumstances. Unfortunately, there is no single test for convergence
which is useful under all conditions.
The Ratio Test determines if the terms of a given series are approaching 0 at a
rate sufficient for convergence by considering the ratio between successive
terms of the series.

Objectives: After a successful accomplishment of this lesson the student will


be able to:
 State the Ratio Test.
 Use the Ratio Test to determine convergence or divergence of a given series.
 Explain the advantages of using Ratio Test.

Theorem 2.8 (Ratio Test): Let ∑ an be a series with a n > 0 for every n and
n =1

suppose that
a n +1
lim = L (possibly ∞ )
n →∞ a
n


i) If 0 ≤ L < 1 , then ∑ an converges
n =1


ii) If L > 1 , then ∑ an diverges
n =1

iii) If L = 1 , then from this test alone we cannot draw any conclusion about the

convergence or divergence of ∑ an .
n =1

Proof: (i) First we assume that 0 ≤ L < 1 , and we let S be a number such that
L < S < 1 . Since
a n +1
lim = L and L < S
n →∞ a
n

66
Applied Mathematics II

a n +1
There is an integer N such that for n ≥ N we have ≤ S or alternatively,
an

a n +1 ≤ a n S . By letting n = N we obtain a N +1 ≤ a N S and then by letting n = N + 1 we

obtain
a N + 2 ≤ a N +1 S ≤ (a N S ) S = a N S 2

In general, for any positive integer n we find that

0 < a N + n ≤ a N + n −1 S ≤ a N + n − 2 S 2 ≤ L ≤ a N +1 S n −1 ≤ a N S n …………………..(1)

Since 0 < S < 1 , the geometric series ∑ aN S n converges.
n =0


From (1) and the Comparison Test we conclude that ∑ a N +n ,which is the same
n =1

∞ ∞
series as ∑ an also converges. From the convergence of ∑ an we conclude
n =N n =N


that ∑ an converges.
n =1

(ii) Let S be a number such that L > S > 1 .There is an integer N such
a n +1
that for n ≥ N we have ≥ S , or equivalently a n +1 ≥ a n S .
an

By letting n = N we obtain a N +1 ≥ a N S , and then by letting n = N + 1 we obtain

a N + 2 ≥ a N +1 S ≥ (a N S ) S = a N S 2 .

In general for any positive integer we find that


a N + n ≥ a N + n −1 S ≥ a N + n − 2 S 2 ≥ L ≥ a N +1 S n −1 ≥ a N S n > 0

Since S > 1 the geometric series ∑ aN S n diverges and by Comparison Test
n =0


∑ a N +n also diverges.
n =1

When do you use the Ratio Test? Ratios are fractions, and they tend to simplify
nicely if the top and bottom contain products or powers. For example, if the nth

67
Applied Mathematics II

term of the series contains factorials, you ought to give the Ratio Test serious
consideration.

The examples below will show that the ration test is inclusive if L = 1 . Namely,
the third example considers a divergent series for which L = 1 and the fourth
example considers a convergent series for which L = 1 . Hence some other test
will be necessary to determine the behavior of any series for which the ratio test
yields L = 1 .

n
Example 1. For the series ∑ 3n
n =1

n
if we let a n = ,
3n
n = 1, 2, 3, K , then
n +1
 n + 1  3  1
n
a n +1 3 n +1  1 1
L = lim = lim = lim   n +1  = lim 1 +  =
n →∞ a
n
n →∞ n n → ∞  n  3  3 n → ∞  n 3
n
3
Thus 0 ≤ L < 1 and the series converges by Ratio test.

5n
Example 2. For the series ∑n+2,
n =1

5n
if we let a n = ,
n+2
n = 1, 2, 3, K , then

5 n +1 2
1+
 n + 2  5 
n +1
a n +1 n+3 n =5
L = lim = lim = lim    = 5 lim
n →∞ a 5n n →∞ n + 3  5 n  3
n
n →∞
  n →∞
1+
n+2 n
Thus L > 1 and the series divergence by the ratio test.

1
Example 3. For the harmonic series ∑n
n =1

The ratio test yields

68
Applied Mathematics II

1
L = lim n + 1 = lim n = lim 1 = 1
n→∞ 1 n →∞ n + 1 n →∞ 1
1+
n n
This shows that it is possible for the series to diverge when L = 1 .

1
Example 4. For the convergent p-series ∑ n2 ,
n =1

the ratio test yields


1 2
 
2  
(n + 1)2 n2  n  1
L = lim = lim = lim   = lim   = 1.
n →∞ 1 n →∞ (n + 1)2 n →∞ n + 1  n →∞  1
1 + 
n2  n
This shows that it is possible for a series to converge when L = 1 .

3n
Example 5. For the series ∑ n! ,
n =1

3n
if we let a n = ,
n!
n = 1, 2, 3, K , then

3 n +1
an + 1 (n + 1)!  n !   3 n +1  3
L = lim = lim = lim    n  = lim =0
3n n →∞ (n + 1)!
n →∞ a
n
n →∞
   3  n →∞ n + 1
n!
Thus 0 ≤ L < 1 and the series converges by ratio test.
Key terms
• Ratio test
Self test.
1. Determine whether the following series converges or diverges.
∞ ∞ ∞ ∞
n! n! (2n)! 1.3.5.7....(2n − 1)
a) ∑2
n =1
n
n
b) ∑ (2n)!
n =1
c) ∑ n!(2n)
n =1
n
d) ∑
n =1 2.4.6.8.....(2 n) n
n n

69
Applied Mathematics II


2. Construct a non negative series ∑a
n =1
nn such that lim n an doesn’t exist but
n →∞

∑a
n =1
nn diverges.


an +1
3. Construct a non negative series ∑a
n =1
n such that lim
n →∞ an
doesn’t exist


but ∑a n =1
n diverges.

Lesson 2.8

Lesson title: Root Test


Introduction: The Ratio Test discussed in the previous lesson helps us to test
convergence or divergence of a series with out taking another series with which
we compare the given series. Here in this section also we will see another test
which can be applied without considering other series whose convergence or
divergence is known. The test that we are going to see is called the Root Test.
The Root Test is similar to the Ratio Test but in this test instead of taking the
limit of successive quotients of terms, we take the limits of roots of terms.

Objectives: At the end of this lesson the student will be able to:
 State the Root Test.
 Use the Root Test to test convergence or divergence of series.
 Tell the conditions where the Root Test cannot be applied to test
convergence.

Theorem 2.9 (Root Test) Let ∑ an be a non-negative series and assume that
n =1

lim n a n = L ( possibly ∞ )
n →∞


i) If 0 ≤ L < 1 , then ∑ an converges.
n =1

70
Applied Mathematics II


ii) If L > 1 , then ∑ an diverges
n =1

iii) If L = 1 , then from this test along we cannot draw any conclusion about the

convergence or divergence of ∑ an .
n =1

Proof: i) Assume that 0 ≤ L < 1 and let S be any number such that L < S < 1 .
Since lim n a n = L and L < S
n →∞

there is an integer N such that for n ≥ N we have n a n ≤ S , or equivalently,



n
a n ≤ S . Since S < 1 , the geometric series ∑Sn converges. Then by Comparison
n =N

∞ ∞
Test the series ∑ a n converges, and hence ∑ an also converges.
n =N n =1

ii) It can be proved analogously.


You might consider using the Root Test if the general term of the series has lots
of nth powers, since these will simplify when you take the nth root.
Activity 1. Prove part (ii) of Theorem 2.6.
∞ n
 2n + 5 
Example 1. Test the convergence of the series ∑   .
n =1  3n + 4 

n
 2n + 5 
Solution: Let a n =   .
 3n + 4 
5
2+
2n + 5 n
Then n an = =
3n + 4 4
3+
n
5
2+
Now lim n a n = lim n = 2 < 1.
n →∞ n →∞ 4 3
3+
n
∞ n
 2n + 5 
Thus the series ∑   converges by Root Test.
n =1  3n + 4 

71
Applied Mathematics II


3n
Example 2. Does the series ∑ 2n
converge or diverge?
n =1

3n 3n 3
Solution: Let a n = . So n an = n = .
2n 2n 2

3 3
Now lim n a = lim = <1.
n →∞ n →∞ 2 2
Therefore ,the series converges by the Root Test.

2n
Activity 2. Does the series ∑ 3n converge or diverge?
n =1

∞ n2
1 n
Example 3. Test the convergence or divergence of the series ∑ 2 1 −  .
n =1  n
n2
n
1
Solution: Let a n = 2 1 −  .
 n
1  
 n 
2 n
n  
 n 1   1  1 
lim an = lim 2 1 − 
n = lim 21 −  = lim 2
n →∞ 
 n   n →∞  n  n →∞   1  
n →∞ n
Now    1 +  
 n −1 
2
= <1
e
Therefore, by Root Test, the given series converses.
Example 4. Use Root Test to show the convergence of the series
n2
 2
∑ (3 + sin n ) 1 − n  .
n

n2
 2
Solution: Letting a n = (3 + sin n ) n
1 −  , we have
 n
1
 n2 n
 2
lim n a n = lim (3 + sin n )n 1 −  
n →∞ n →∞   n 
 

72
Applied Mathematics II

n
 2
= lim (3 + sin n ) 1 − 
n→ ∞  n
n
 2 4
≤ lim 41 −  = 2 < 1
n →∞  n e
Therefore the series is convergent.
Key terms
• Root test
Self test.
1. Determine the convergence or divergence of the following series.
∞ ∞ ∞
ln n π n
a) ∑
n =1 e
n
b) ∑ n( 4 )
n =1
n
c) ∑ ( 2n + 5 )
n =1
n

 0 for n even


∑a
n
2. Let be a series with an =  n  . Is this series
n =1
n
 
 2n + 1  , for n odd

convergent or divergent series?

Lessons 2.9& 2.10

Lesson title: Series with both positive and negative terms


Introduction: In the previous sections we have limited our study of series to
those series having non-negative terms only. However, there are series which
contain negative as well as positive terms, for such series we have to develop
techniques to determine their convergence. The convergence tests investigated
so far apply to series with nonnegative terms only. In this section, we will learn
how to deal with series that may have negative terms.

Objectives: Upon the successful accomplishment of this lesson the student will
be able to:
 Test the convergence or divergence of any given series.
 Explain the differences between conditional convergence and absolute
convergence.

73
Applied Mathematics II

 Use the series tests so far discussed to test absolute convergence and hence
convergence.

Absolute and Conditional Convergence


∞ ∞
Definition: The series ∑ a n is said to converge absolutely if the series ∑ an
n =1 n =1

∞ ∞
converges. If the series ∑ a n converges and ∑ an diverges , then we call the
n =1 n =1


∑ an is conditionally convergent.
n =1

Notation: It is common to use the abbreviations A.C. or abs. cgt for absolutely
convergent.
Example 1. The series

(− 1)n+1 = 1 1 1 1 ∞
(− 1)n+1 ∞
1
∑ n 2
1 2

2 2
+
3 2

4 2
+ L converges absolutely because ∑ n 2
=∑
n2
n =1 n =1 n =1

converges.
Theorem 2.10: If a series ∑ an is absolutely convergent, then it is convergent.

(or equivalently; if ∑ an is convergent, then ∑ an converges.)

Proof: Observe that the inequality


− an ≤ an ≤ an …………………………………(1)

is true.
If we add a n to each side of this inequality, we get

0 ≤ an + an ≤ 2 an

Let bn = a n + a n .Then 0 ≤ bn ≤ 2a n . If ∑ an is absolutely convergent, then ∑ an is

convergent, so ∑ 2 an is convergent. Therefore ∑ bn is convergent by

comparison Test. Since a n = bn − a n is convergent,

∑ a n = ∑ bn − ∑ a n is convergent.

74
Applied Mathematics II

Remark. We can use the fact from Theorem 2.7 to show that a series converges
in situations where other tests are difficult to apply.
Example 2. The series

sin n sin 1 sin 2 sin 3
∑ 2n
=
2
+ 2 + 3 +L
2 2
n =1

does not alternate. In fact,


sin 1 ≈ 0.84147 , sin 2 ≈ 0.90930 , sin 3 ≈ 0.14112 , sin 4 ≈ − 0.75680 , K
Thus, you can't use the Alternating Series Test. On the other hand, since the
series has negative terms, many convergence tests - the Integral Test, the Ratio
Test, the Root Test - don't apply (why?).
∞ sin n
The trick is to consider the absolute value series, which is ∑ n .
n =1 2
Since sin n ≤ 1 for all n ,
sin n 1
n
≤ .
2 2n

1
We know that the series ∑ 2n is a convergent geometric series. Therefore, the
n =1
∞ sin n
series ∑ 2n
converges by Comparison Test.
n =1
∞ ∞
sin n sin n
Thus, the original series ∑ 2 n
converges absolutely. Hence, the series ∑ 2n
n =1 n =1
converges.

sin n
Example 3. Is ∑ n2
convergent or divergent?
n =1
sin n ∞
sin n 1 1
Solution: Put a n =
n 2
. Then 0 ≤ a n =
n 2

n 2
, and we Know that ∑ n2 is
n =1


convergent, so ∑ an is convergent by the Comparison Test.
n =1


Hence ∑ an is absolutely convergent, so it is convergent.
n =1


sin n
That is, ∑ n2
is convergent.
n =1

75
Applied Mathematics II

The following is a very slight rewarding of the Comparison Test, to mention


absolute convergence:
In problems which ask you to check for absolute or conditional convergence,
you should be careful to do things in the correct order. Here is how to approach
the question: “ Does the series converge absolutely, converge conditionally, or
diverge?”
1. Scan the series quickly and see if you can apply the Divergence Test. If
lim a n ≠ 0 , the series diverges (and that is all you have to do).
n →∞


2. Check the absolute values series ∑ an for convergence using the
n =1

convergence tests for non-negative series. If it converges, the original


series converges absolutely and you can stop. If it diverges, go on to the
next step
3. Now you know that the absolute value series diverges, you need to check
for conditional convergence. If it converges, you conclude that it
converges conditionally; otherwise, it diverges.
Example 4. Does the series

2n 2 + 3
∑ (− 1) n

3n 2 − 1

converge absolutely, converge conditionally, or diverge?


Solution: Notice that

2n 2 + 3 2 2
n 2n + 3
lim = , so lim (− 1) is undefined.
n →∞ 3n 2 − 1 3 n →∞ 3n 2 − 1
Therefore, the series diverges by Divergence Test.
Example 5. Does the series

(− 1)n
∑ 1  1 
n =1
 n 2 + 1 n 3 + 1
  
  
converge absolutely, converge conditionally, or diverge?
Solution: We can see that

76
Applied Mathematics II

1
lim =0,
n →∞  1  1 
n2 + 1 n 3 + 1
  
  
so the Divergence Test doesn’t work.

1
Consider the absolute value of the series ∑ 1  1 
. (Taking absolute
n =1
 n 2 + 1 n 3 + 1
  
  

values removes the (− 1)n .)


For large n ,
1 1 1
≈ 1 1 = 5 .
 1  1 
n2 + 1 n 3 + 1 n 2 ⋅ n 3 n 6
  
  
Applying Limit Comparison:
1
 1  1 
n2 + 1 n 3 + 1 5
  
   n 6
lim = lim =1 .
n →∞ 1 n→ ∞ 
1  1 
5  n 2 + 1 n 3 + 1
  
n6   
The limit is finite and positive.

1 5
The series ∑ 5
diverges, because it is a p-series with p = < 1 . Therefore, the
6
n =1
n6
absolute value series diverges by Limit Comparison Test, and the original series
does not converge absolutely.
1
Now return to the original series. The terms alternate. If f ( x) = ,
 1  1 
x2 + 1 x 3 + 1
  
  
then

77
Applied Mathematics II

1 2
1 −2 1 −3
x x
f ' ( x) = − 2 − 3
2 2
 1   1   1  1 
 x 2 + 1  x 3 + 1  x 2 + 1 x 3 + 1
      
      
f ' ( x) < 0 for x ≥ 1 , so the terms decrease in magnitude. Finally,
1
lim =0.
n →∞ 
1  1 
 n 2 + 1 n 3 + 1
  
  
The hypotheses of the Alternating Series Test are satisfied, so the original series
converges. Since the original series converges, but does not converge
absolutely, it converges conditionally.
Example 6. Does the series

n
∑ (− 1)n n 3 + 2
n =1

converge absolutely, converge conditionally, or diverge?



n
Solution: Consider the absolute value series ∑ n 3 + 2 . Apply Limit Comparison
n =1

Test:
n
3 3
n +2 = n
lim lim 3
=1
n →∞ 1 n →∞ n + 2
n2

1
The limit is a finite positive number. The series ∑ n2 converges because it is a
n =1

p-series with p = 2 > 1 .



n
Therefore, ∑ n3 + 2 converges by Limit Comparison Test. Hence the series
n =1


n
∑ (− 1)n n 3 + 2 converges absolutely. Therefore the series is convergent.
n =1

Example 7. Does the series

78
Applied Mathematics II

πn πn
∞ sin + cos
∑ 3
n 2
4
n =1

converge absolutely, converge conditionally, or diverge?


πn πn
Solution: The table below shows the sign of sin + cos for n = 1 to n = 10 .
3 4

This is not an alternating series


πn πn

sin + cos
3 4
Consider the absolute value series ∑ n2
.
n =1

πn πn
sin + cos
πn πn 3 4 2
sin + cos ≤ 2 , so 2
≤ .
3 4 n n2

2
∑ n2 converges, since it is a multiple of a p-series with p = 2 > 1 .
n =1

πn πn

sin + cos
3 4
Therefore, the absolute value series ∑ n2
converges by Comparison
n =1

Test. Hence the original series converges absolutely.

Theorem 2.11: If the sequences {a n } and {bn } satisfy

i) a n ≤ bn for all n ; and



ii) ∑ bn is convergent;
n =1

then is absolutely convergent (and hence convergent).

The previous theorem says that any series which converges absolutely also
converges. We shall see later that the converse of this statement does not hold;
namely, there are series which converge, but do not converge absolutely.

79
Applied Mathematics II

Example 8. The series



(− 1)n+1 = 1 − 1 + 1 − 1 + 1 − 1 + L ,
∑ n 2 3 4 5 6
n =1

known as the alternating harmonic series, is not absolutely convergent since



(− 1)n+1 ∞
1
∑ n
=∑
n =1 n =1 n

is the harmonic series, which diverges. Hence the previous Theorem does not
provide any information on the behavior of the alternating harmonic series
itself. We shall see below that in fact the alternating harmonic series converges
even though it is not absolutely convergent.
Remark. If you are testing for absolute convergence, all the techniques for
positive series are applicable.
∞ ∞
Definition: ∑ a n is said to be conditionally convergent if ∑ an convergent but
n =1 n =1


∑ an diverges.
n =1

Theorem 2.12: Every series is either absolutely convergent or conditionally


convergent or divergent.
Activity. Prove theorem2.9.
Key terms
• Absolute convergence
• Conditional convergence
Self Test.
∞ ∞
1. If ∑ an
n =1
is absolutely convergent, must ∑ (a
n =1
n + an +1 ) be absolutely

convergent? Explain your answer.


2 .Classify the series as absolutely convergent, conditionally convergent,
divergent
∞ ∞ ∞
1 1.3.5.7....(2n + 1) 1
a) ∑ (−1) n +1
n =1
1
b) ∑ (−1) n
n =1 2.5.8.....(3n + 2)
c) ∑ (−1)
n =1
+1n

n(ln n) 2
nn

80
Applied Mathematics II

Lesson 2.11

Lesson title: Alternating Series


Introduction: In general, determining whether a series is not absolutely
convergent or divergent is a difficult problem. However there is one type of
series for which we have, under certain conditions, a simple test. These series
are alternating series, the series which, like those in the previous examples,
alternate in sign from one term to the next.
If a series has only positive terms, the partial sums get larger and larger. If they
get large too rapidly, the series will diverge. However, if some of the terms are
negative, the negative terms may cancel with the positive terms and prevent the
partial sum from “blowing up”.

Objectives: After a successful completion of this lesson the student will be able
to:
 Identify alternating series.
 State conditions for convergence of an alternating series to converge.
 Test convergence of alternating series.

Definition: A series in which the terms are alternatively positive and negative is
called an alternating series. That is an alternating series is of the form

∑ (− 1)n+1 a n = a1 − a 2 + a3 − a 4 + L , or
n =1


∑ (− 1)n a n = − a1 + a 2 − a3 + a 4 − L
n =1

with each a n > 0 .


∞ ∞
Theorem 2.13 ( The alternating series test): Let ∑ (− 1)n+1 a n ( or ∑ (− 1)n a n )
n =1 n =1

be an alternating series. Suppose that

81
Applied Mathematics II

(i) a n > 0 for all n .

(ii) {an }n =1 is monotone decreasing ( i.e. , a n ≥ a n +1 for all n ) , and


(iii) lim a n = 0 .
n →∞

∞ ∞
Then ∑ (− 1)n+1 a n ( respectively ∑ (− 1)n a n ) is convergent.
n =1 n =1


Proof: We look at the partial sums {S n } of the series ∑ (− 1)n+1 a n . Now,
n =1

S 2 n = a1 − a 2 + a 3 − a 4 + L + a 2 n −1 − a 2 n
≤ a1 − a 2 + a 3 − a 4 + L + a 2 n −1 − a 2 n + a 2 n +1 − a 2 n − 2 = S 2 n + 2

Thus {S 2 n } is monotone increasing. Also,

S 2 n = a1 − a 2 + a 3 − a 4 + a5 − a 6 + L + a 2 n −1 − a 2 n ≤ a1
1424 3 1424 3

Thus {S 2 n } is bounded above by a1 . Then by Monotone Convergence Theorem,

{S 2n } is convergent, and write lim S 2 n = S . For any ε > 0 , there exists N 1 and
n →∞

ε ε
N2 such that S 2 n − S < for n > N 1 and a n − 0 < for n > N 2 . Choose
2 2
N = max{2 N 1 + 1, N 2 } . We show that S n − S < ε for all n > N .

N
Case I n = 2k with n > N . Then k > > N1 .
2
ε
Then S 2 k − S < < ε by the construction of N 1 .
2
N −1
Case II n = 2k + 1 with n > N . Then k > ≥ N 1 and n = 2k + 1 > N ≥ N 2 .
2
Thus
ε ε
S 2k − S < and a 2 k +1 <
2 2
Now
ε ε
S n − S = S 2 k +1 − S = S 2 k + a 2 k +1 − S ≤ S 2 k − S + a 2 k +1 < + .
2 2

82
Applied Mathematics II

Thus Sn − S < ε for all n>N and so lim S n = S (convergent), and thus
n →∞


∑ (− 1)n+1 a n is convergent.
n =1

Example 1. Show convergence or divergence of the series



1
∑ (− 1)n n p
n =1

Solution
1
Case(i): If p ≤ 0 , then the n th term (− 1)n does not tend to 0. Thus the series
np
diverges in this case by The Divergence Test.
1
Case (ii): Assume that p > 0 . Let a n = . Then a n > 0 , monotone decreasing
np
and lim a n = 0 .Thus the alternating series converges in this case.
n →∞


1
In conclusion we have that the series ∑ (− 1)n n p converges when p > 0 and
n =1

diverges when p ≤ 0 .

1
Example 2. Does ∑ (− 1)n n − n
converge or diverge?
n=2

Solution: Notice that the series alternates, and


1 1
= lim
lim
n →∞ n− n n→ ∞ n ( n −1 )=0 .
1
1 −2
x 1−
1 2
Set f ( x) = . Then f ' ( x) = − ,
x− x x− x
2
( )
So f ' ( x) < 0 for x ≥ 2 . It follows that the terms of the series decrease in absolute
value. By the Alternating Series Test, the series converges.

cos πn
Example 3. Does the series ∑ n +1
converge or diverge?
n =0

83
Applied Mathematics II

Solution: This is an alternating series, although it is disguised by the absence of


n +1
the usual (− 1) or (− 1)
n
. But notice that
cos 0 = 1 , cos π = − 1 , cos 2π = 1 , cos 3π = − 1 , K .

(− 1)n
In fact, cos πn = (− 1) , so the series can be written as
n
∑ n +1
.
n =0

1
The series alternates. If f ( x) = , then
x +1
1
f ' ( x) = − 3
,
2( x + 1) 2

which is always negative. Hence, the terms decrease in magnitude.


Finally,
1
lim =0.
n →∞ n +1
The conditions of the Alternating Series Test are satisfied. Hence, the series
converges.
A partial sum S n of any convergent series can be used as an approximation to

the total sum S but this is not of much use unless we can estimate the
accuracy of the approximation. The error involved in using S ≈ S n is S − S n . The

next theorem tells us how small the error will be if we take S n as an

approximation for S .

Theorem 2.14: If S = ∑ (−1) n −1 a n is the sum of an alternating sequence that

satisfies
a) 0 ≤ a n +1 ≤ a n and

b) lim a n = 0 , then
n →∞

S − S n ≤ a n +1

Proof: The idea is similar to the one for the proof of the Alternating Series Test.
We have

S − S n = (− 1)n a n +1 + (− 1)n +1 a n + 2 + (− 1)n + 2 a n + 3 + L

84
Applied Mathematics II

= (− 1)n [a n +1 − a n + 2 + a n +3 − L]

And so
S − S n = (a n +1 − a a + 2 ) + (a n + 3 − a n+ 4 ) + L

= a n +1 − (a n + 2 − a n + 3 ) − (a n + 4 − a n + 5 ) − L

Every term in brackets is positive, so S − S n ≤ a n +1 .

Example 4. For the alternating harmonic series, if

S =∑

(− 1)n+1
n =1 n
And
n +1
1 1 1
S n =1 − + − + L +
(− 1) n
=∑
(− 1)k +1 ,
2 3 4 n k =1 k
Then
1
S − Sn ≤ , for n = 1, 2, 3 , K .
n +1
For example
1 1 1 1 1
S100 = 1 − + − + L + − = 0.688172 , so
2 3 4 99 100
1
S − Sn ≤ = 0.009901 ,
101
Where both results have been rounded to 6 decimal places. In other words, the
sum of the alternating harmonic series differs from 0.688172 by less than
0.009901. In fact, since the next term in the series is positive, we know that S
must lie between 0.688172 and
0.688172 + 0.009901 = 0.698073.
Activity 1. (a) Approximate

S =∑

(− 1)n
n =0 n!

using S15 = ∑
15
(− 1)n
n =0 n!

(b) Find an upper bound for the error in approximating S by S15 .

85
Applied Mathematics II

(c) Find the smallest n such that the absolute value of the error in
approximating S by

Sn = ∑
n
(− 1)k
k =0 k!
Is less that 0.000001. What is this approximation?
Activity 2. (a) Approximate

S =∑

(− 1)n+1
n =1 n 2n

by S 50 = ∑
50
(− 1)k +1 .
k =1 k 2k
(b) Find an upper bound for the absolute value of the error in
approximating S by S 50 .

(c) Find the smallest n such that the absolute value of the error in
approximating S by

Sn = ∑
n
(− 1)k +1
k =1 k 2k
is less than 0.0001. What is this approximation?

Key terms
• Alternating series
• Alternating series test
Self Test.
2. Determine whether the series converges or diverges .

n +1 ∞
(ln n) 2 ∞
n!
a) ∑ (−1)n
n =1 4n
b) ∑ (−1) n
n =1 n
c) ∑ (−1)
n =1
n +1

100 n

π 1 
d) ∑ (−1)
n =1
n
cot  − 
 2 n

1
3. For which positive values of p does the series ∑ (−1)
n =1
n

np
converge?

4. Approximate the sum of the given series with an error less than 0.001.

86
Applied Mathematics II

∞ ∞
1 8
a) ∑ (−1) n +1
n =1
2
6n + n + 1
b) ∑ (−1)
n =1
n
n
10 + 1

Lesson 2.12

Lesson title: The Generalized Convergence Tests


Introduction: So far we have been dealing with non-negative series or a special
type of series like alternating series only. We have number of theorems
applicable to non-negative series. But all series we get from real problems are
not these types only. So we have to make ourselves ready to deal with all types
of series. Theorem 2.7 combined with all the tests for positive series, we obtain
convergence tests that apply to any series, positive or not.
Objectives: At the end of this lesson the student will be able to:
 Use the generalized convergence tests to Test convergence or divergence of
series.
 Extend all the tests for positive series to test convergence of series which
are not positive.
 State all the generalized convergence tests.

Theorem 2.15( Generalized Convergence Tests): Let ∑ an be series.
n =1


Generalized Comparison Test. If a n ≤ bn for n ≥ 1 , and if ∑ bn converges,
n =1


then ∑ an converges (absolutely).
n =1

an
Generalized limit comparison Test. If lim = L , where L is a positive number,
n →∞ bn
∞ ∞
and if ∑ bn converges, then ∑ an converges (absolutely).
n =1 n =1

87
Applied Mathematics II

Generalized Ratio Test (D’Alembert’s Ratio Test): Consider an infinite series



a n +1
∑ an such that a n ≠ 0 for n ≥ 1 , and look at lim
n →∞ an
=r .
n =1


a n +1
i) If this limit is infinity or lim
n →∞ an
> 1 , then the series ∑ an is divergent.
n =1

an +1
ii) If the limit exists and is (strictly) less than 1 ( that is, 0 ≤ lim <1 ),
n →∞ an

then the series ∑ an is convergent.
n =1

a n +1
Remark: What happens if lim = 1 ? The answer is we don’t know! In this
n →∞ an

situation, you need to apply other method.

Generalized Root Test. Suppose that lim n a n = r (possibly ∞ ).


n →∞

∞ ∞
i) If 0 ≤ r <1 , then ∑ a n converges (absolutely). ii) If r >1 , then ∑ an diverges.
n =1 n =1

If r = 1 , then from this test alone we cannot draw any conclusion about the
convergence of the series.
Activity 1. Prove Theorem 2.9.
Example 1. Determine if the following series is convergent or divergent.

(−10) n
∑ 4 2n+1 (n + 1)
n =1

(−10) n
Solution: Notice that the n th term a n = ≠ 0 , so we can use the
4 2 n +1 (n + 1)
Generalized Ratio Test to determine the convergence or divergence of the series.

(−10) n +1
Now a n +1 = , hence
4 2 n + 3 (n + 2)

a n +1 (−10) n +1 4 2 n +1 (n + 1)
lim = lim
n →∞ an n →∞ ( −10) n 4 2 n + 3 ( n + 2)

88
Applied Mathematics II

− 10(n + 1)
= lim
n→ ∞ 4 2 ( n + 2)

10 n +1
= lim
16 n→∞ n + 2
10
= <1 .
16
So, by Generalized Ratio Test the series is convergent.

1
Example 2. Consider the harmonic series ∑n.
n =1

1 1
For this series a n = and a n +1 = .
n n +1
a n +1 n
Here, = → 1 as n → ∞ . So the Generalized Ratio Test does not tell us
an n +1

anything about the convergence or divergence of this series. But we know that
this series is divergent using other tests.


1
Example 3. Consider the series ∑ n2 .
n =1

1 1
Solution: For this series a n = and a n +1 = .
n 2
(n + 1)2
2
a  n 
We have n +1 =   → 1 as n → ∞ .
an  n + 1
Again the Generalized Ratio Test cannot be applied to find out convergence or
divergence of the series. However, we know this series is a convergent p-series.
From the above two examples you can see that a series may converge or diverge
a n +1
when lim =1 .
n →∞ an

Example 4. Determine if the following series is convergent or divergent.


n
∞  5n − 3n 3 
∑  3 
n =0  7n + 2 

89
Applied Mathematics II

Solution: Now it is more appropriate to use the Generalized Root Test to


determine the convergence or divergence of this series. Here,
n
 5n − 3n 3 
a n =  3
 .

 7 n + 2 
1
n n
 5n − 3n 3  5n − 3n 3 −3 3
So lim  3

 = lim 3
= = < 1 . Which by Generalized Root Test
n →∞
 7n + 2  n→∞ 7n + 2 7 7

implies that the series is convergent.


Key terms
• Generalized comparison test
• Generalized root test
• Generalized ratio test
Self Test.
1. Determine which series diverge , which converge conditionally ,which
converge absolutely .
n

(−1) n ln n ∞
 n+2  ∞
nconπ ∞
(−1) n +1
a) ∑ b) ∑ (−1) n +1
  c) ∑ 2
d) ∑
n =3 n n =3  3n − 1  n =3 n + 1 n =3 n +1 + n

2. Prove that if ∑a
n =1
n converges absolutely , then

∞ ∞

∑ an ≤ ∑ an .
n =1 n =1

Summary
Now we have got all of our tests out of the way. It is time to think about
organizing all of them into a general set of guidelines to help us determine the
convergence of a series.
Since more than one test may apply, you should always go through the
guidelines and identify all possible tests that can be used on a given series.

90
Applied Mathematics II

Once this has been done you can identify the test that you feel will be the
easiest for you to use.

Set of guidelines.
With a quick glance, does it look like the series terms don’t converge to zero? If
so, use the Divergence Test.
∞ ∞
1
1. Is the series a p-series ( ∑ p
) or a geometric series ( ∑ ar n or ∑ ar n−1 ) ? If
n n =0 n =1

so, use the fact that p-series will only converge if p>1 and a geometric
series will only converge if r < 1 . Remember as well that often some

algebraic manipulation is required to get a geometric series into the


correct form.
2. Is the series similar to a p-series or a geometric series? If so, try the
Comparison Test.
3. Is the series a rational expression involving only polynomials under
radicals (i.e. a fraction involving only polynomials or polynomials under
radicals)? If so, try the comparison Test and/or the Limit Comparison
Test. Remember however, that in order to use the Comparison Test and
the Limit Comparison Test the all series terms need to be positive.
4. Does the series contain factorials or constants raised to powers involving
n ? If so, then the Ratio Test may work. Note that if the series terms
contain a factorial then the only test that we have got that will work is the
Ratio Test.
5. Can the series terms be written in the form a n = (−1) n bn or a n = (−1) n +1 bn ?

If so, then the Alternating Series Test may work.


6. Can the series terms be written in the form a n = (bn ) n ? If so, the Root Test

may work.

7. If a n = f (n) for some positive, decreasing function and ∫ f ( x) dx is easy to
m

evaluate then the Integral Test may work.

91
Applied Mathematics II

8. Again, remember that these are only a set of guidelines and not a set of
hard and fast rules to use when trying to determine the best test to use
on a series.

Next, we present the tests we have so far discovered as follows:

Divergence Test
If lim a n ≠ 0 , then
n →∞
∑ an will diverge.

Integral Test. Let {an }



n=m be a series of positive terms.

Suppose that f (x) is a positive, decreasing function on an interval [ m, ∞) and

that f (n) = a n then,


∞ ∞
If ∫ f ( x)dx is convergent so is ∑ an .
m n =m

∞ ∞
If ∫ f ( x)dx is divergent so is ∑ an .
m n =m

Comparison Test
∞ ∞
Suppose that we have two series ∑ a n and ∑a
n =m
n with an , bn > 0 for all n and
n =m

0 < an < bn for all n . Then,

i) If ∑ bn is convergent then so is ∑ an .
ii) If ∑ an is divergent then so is ∑ bn .

Limit Comparison Test


Suppose that we have two series ∑ an and ∑ bn with an , bn > 0 for all n . Define,

an
lim =c
n →∞ b
n

92
Applied Mathematics II

If c is positive (i.e. c > 0 ) and is finite, then either both series converge or both
series diverge.

Alternating Series Test


Suppose that we have a series ∑ an and either a n = (−1) n bn or a n = (−1) n +1 bn

where bn ≥ 0 for all n . Then if,

lim bn = 0 and, {bn } is a decreasing sequence ,then the series


n →∞
∑ an is convergent.

Generalized Ratio Test


Suppose we have the series ∑ a n . Define,
a n +1
L = lim
n →∞ an

Then,
i) If L < 1 the series absolutely convergent (and hence convergent).
ii) If L > 1 the series is divergent.
iii) If L = 1 the series may be convergent or divergent.

Generalized Root Test


Suppose that we have the series ∑ a n . Define,
1
L = lim n a n = lim a n n
n →∞ n →∞

Then,
if L < 1 the series is absolutely convergent (and hence convergent).
if L > 1 the series is divergent.
if L = 1 the series may diverge or converge.

• Given a series ∑ ak its n th partial sum S n is given by
k =1

93
Applied Mathematics II

n
S n = ∑ a k = a1 + a 2 + a 3 + L + a n
k =1


The sequence {S n } is called the sequence of partial sums of the series ∑ ak .
k =1

• Consider the sequence of partial sums {S n } of the series ∑ ak .If this
k =1


sequence converges to a number S , then we say that the series ∑ ak
k =1

converges to S and write



∑ a k = nlim
→∞
Sn = S
k =1

On the other hand if the sequence of partial sums {S n } diverges, then we say

the series ∑ ak diverges.
k =1

• Series of the form



∑ ar n = a + ar + ar 2 + ar 3 + L , where a ≠ 0 are Geometric series
n =0


The geometric series ∑ ar n converges if and only if − 1 < r < 1 , and,
n =0


a
∑ ar n = 1 − r for − 1 < r < 1 .
n =0


• If {a n } and {bn } are convergent infinite series such that ∑ an = A and
n =1


∑ bn = B . Then
n =1

∞ ∞ ∞
1. ∑ (a k + bk ) = ∑ a k + ∑ bk = A + B
k =1 k =1 k =1

∞ ∞ ∞
2. ∑ (a k − bk ) = ∑ a k − ∑ bk = A − B
k =1 k =1 k =1

94
Applied Mathematics II

∞ ∞
3. ∑ ca k = c∑ a k = cA if c is any real constant.
k =1 k =1

• If ∑ ak converges, then lim a k = 0 .
k →∞
k =1

• If lim a k ≠ 0 (or does not exist), then
k →∞
∑ ak diverges.
k =1


1 1 1 1 1
• The harmonic series ∑ n = 1 + 2 + 3 + 4 + L. For this series lim a n = lim
n →∞ n →∞ n
=0.
n =1


1
But the series ∑n diverges.
n =1

Review Exercises
1. Determine whether or not the series converges, and if so find its sum.

8 4  ∞
n ∞
1
a) ∑  3 n + (n + 3)(n + 4)  b) ∑ c) ∑ n(n + 2)
n =1   n =1 1 + n2 n =1


1
d) ∑ n(n + 1)(n + 2)
n =1

2. Test the following series for convergence or divergence.


1

2 + (−1) n ∞
2n ∞
1
a) ∑ 4 n
b) ∑ n2 c) ∑ n(ln n) 2
n =1 n =1 n =2

n2

n2 ∞
 1 ∞
ln n ∞
1
d) ∑ 3n e) ∑ 2 1 −  n
f) ∑ (−1) n g) ∑ (ln n) ln n
n =1 n =1  n n=2 n n =2

3. For each of the following infinite series, decide whether the series converges
or diverges and explain your answer.

n 2n ∞
a) ∑ 2n b) ∑
n =1 n =1 n

5n

7 n+ 2

c) ∑ d) ∑
n =1 n ! n =1 ( n + 1) !

n2
∞ ∞
4
e) ∑ f) ∑ n 2n
n =1 n ! n =1

95
Applied Mathematics II


1 ∞  3 3n 
g) ∑ 4n − 2
h) ∑  5 n − 5 n 

n =1 n =1  

4. Suppose the terms of the series ∑ an satisfy the difference equation
n =1

(n + 1) a n
a n +1 =
2n
with a1 = 10 . Does this series converge or diverge? Explain. [An equation
which expresses a value of a sequence as a function of the other terms in the
sequence is called a difference equation.]

5. For each of the following infinite series, answer the questions: Does the series
converge absolutely? Does the series converge conditionally? Does the series
converge?

(−1) n n ! ∞
(−3) n
a) ∑ 2n
b) ∑
n =1 n =0 n !


(−1) n π 2 n ∞
3n 2 − 1
c) ∑
( 2 n) !
d) ∑ 4n 2 + 2
n =0 n =1

Unit Assesment:

• Assignment for submission


• Quiz
• Active participation in group discussion
• Individual and group presentation
• Mid examination
• Final examination

References:
1. Engineering Mathematics, Satsty,S.S(2001),2nd edition.
2. Howard Anton,(1980), Calculus with Analytic Geometry
3. Larson R.E and Hoster R.P,(1979),Calculus with Analytic geometry

96
Applied Mathematics II

4. James Starwart
5. Calculus and analytic geometry ,Sherman K. Stein , Anthony Barcellos,
Fifth Edition.

97
Applied Mathematics II

Chapter 3

Power Series
Introduction: We have spent quite a bit of time talking about series in Chapter
2 and with only a couple of exceptions we have spent most of that time talking
about how to determine convergence or divergence of a series. In this chapter
will see a special type of series called power series.
Clearly the rational operations of arithmetic are addition, subtraction,
multiplication, and division. Using only these simple operations, we can
evaluate any rational function of x . But other functions, such as ln x , e x , sin x ,
and so on, cannot be evaluated so simply. Of course, these functions are so
important that their values have been computed and the results printed in
mathematical tables. But we may wonder how the tables were computed. The
answer is: from power series.

Objective: After a successful accomplishment of this chapter, the student will


be able to:
 Define a power series in x.
 Determine the radius of convergence and interval of convergence of a
power series.
 Explain similarities and differences between Taylor series and Maclaurin
series.
 Find Maclaurin series for any given function whenever possible.

Lessons 3.1&3.2

Lesson title: Power series Definitions and Radius of Convergence


Introduction: The first thing to notice about a power series is that it is a
function of x. That is different from any of the series we have seen in the

98
Applied Mathematics II

previous chapter. In chapter 2 we have only allowed numbers in the series but
now we are allowing variables to be in the series as well. However, this will not
change how things work. Everything we know about series holds.
The discussion of the convergence of the power series is still a major question
that we will be dealing with. The different thing that we will see is that, the
convergence of a power series will depend upon the value of x that we put into
the series. A power series may converge for some values of x and not for other
values of x.

Objectives: At the end of this lesson the student will be able to;
 Explain the differences between a real series and a power series.
 Define a power series in x about any constant number c.
 Determine radius and interval of convergence of a given power series.
 Apply the generalized ratio test to determine the radius of convergence of a
power series.

Definition: A power series in x is an expression of the form



∑ an x n , where x is a variable and the an ’s are constants called the
n =0

coefficients of the series.



If a0 = 0 we usually write the power series as ∑ an x n , and if a1 = 0 as well, we
n =1


normally write the series as ∑ an x n . In general the initial index of a power series
n =2

can be any non-negative number. For the sake of uniformity in writing power
series, we will assume that x 0 = 1 for all x , including x = 0 , so that

∑ ak x k = a0 + a1x + a2 x 2 +L .
n=0

For a fixed x, the above series is a series of constants that we can test for
convergence or divergence using the tests discussed so far. A power series may
converge for some values of x and diverge for other values of x.

99
Applied Mathematics II

Definition: A power series in ( x − c) is a series of the form



∑ an ( x − c)n = a0 + a1 ( x − c) + a2 ( x − c)2 +L
n=0

This is referred to as a power series about c. Here we adopt the assumption that
( x − c) 0 = 1 even when x=c.
Definition: i) A power series in x is said to converge at x if the series of real

numbers ∑ an x n converges. Otherwise it is said to diverge at x.
n =0

ii) A power series is said to converge in a set D of real numbers if it


converges for every real number x in D.

Example 1. For what values of x is the series ∑ n! x n convergent?
n =0

Solution: We use the ratio test. Since we have been using a n to mean the

coefficient of x n in this section, we shall use u n to describe the n th term of the

series. Thus u n = n ! x n and, if x ≠ 0 , we have

u n +1 (n + 1) ! x n +1
lim = lim
n →∞ un n→ ∞ n! x n

= lim (n + 1) x = ∞ .
n→∞

Hence, By Generalized Ratio Test, the series diverges when x ≠ 0 . Thus the
given series converges only when x = 0 . Why?

xn
Example 2. For what values of x does the series ∑ converge?
n =0 n !

xn
Solution: This is a power series in x about 0. Note that if we let u n = . For
n!
n = 0, 1, 2 , K , we have

u n +1 n ! x n +1
lim = lim
n →∞ un n →∞ ( n + 1) ! x n

100
Applied Mathematics II

x
= lim =0
n→∞ n +1
Thus, by General Ratio Test the series is absolutely convergent, and hence
convergent, for any value of x. Thus if we define a function, called the
exponential function, by

xn
exp( x) = ∑
n =0 n !

Then this function is defined for all values of x . We shall have much more to
say about this function, which may be thought as the simplest “infinite”
polynomial which is defined for all real numbers.
Notice that the convergence of this function for all x implies, by the nth term
test for divergence that

xn
lim =0
n →∞ n!
for any value of x . We have seen particular cases of this limit in the past, but
this is the first time we have had a simple proof that it is always 0.
∞ ∞
Theorem 3.1: a) If ∑ an s n converges, then ∑ an x n converges absolutely for
n =0 n =0

x<s.
∞ ∞
b) If ∑ an s n diverges, then ∑ an x n diverges for x > s .
n =0 n =0


Proof: To prove (a), let x < s , and rewrite ∑ an x n as follows:
n =0

∞ ∞ n
 x
∑ n ∑ an s n  s 
a x n
=
n=0 n=0


Since ∑ an s n converges by hypothesis, we know that
n =0

lim an s n .
n←∞

101
Applied Mathematics II

Consequently there is a positive integer N such that an s n ≤ 1 for all n > N . This

means that
n n
n n x x
an x = an s ≤ for n > N .
s s

x
Because x < s , which means that <1 . We know that the geometric series
s
∞ n
x
∑ s
converges absolutely. By the Generalized comparison test it follows that
n =N

∞ ∞
∑ an x n converges absolutely, and hence ∑ an x n also converges absolutely.
n= N n =0


To prove (b), we assume that ∑ an s n diverges, and we only need to observe that
n =0


if ∑ an x n were to converge, then reversing the roles of s and x in part (a), we
n =0


would find that ∑ an s n would converge after all, which contradicts our
n =0


assumption. Therefore ∑ an x n diverges.
n =0


Theorem 3.2: For a given power series ∑a
n =0
n ( x − c) n there are only three

possibilities:
1. The series converges only when x = c .
2. The series converges for all x .
3. There is a positive number R such that the series converges if
x − c < R and diverges if x − c > R .

Proof : It is left as an exercise.

Radius of Convergence

For a given power series ∑a n =0
n ( x − c) n there are only three possibilities:

102
Applied Mathematics II

1. The series converges only when x = c .


In this case R = 0
2. The series converges for all x .
In this case R = ∞
3. There is a positive number R (called the radius of convergence) such
that the series converges if x − c < R and diverges if x − c > R .

Remark: Anything can happen for the series ∑ an ( x − c) when x −c =R ,
n=0

absolute convergence, conditional convergence, divergence.

You can use the Ratio Test (and sometimes, the Root Test) to determine the
values for which a Power series converges.
Definition: The set of values of x for which the power series converges is called
the interval of convergence of the power series.
Remark: Imagine a circle centered at the origin of radius R . If x is inside this
circle, then the power series will converge; if x is outside the circle, then the
power series will diverge.
• A power series converges absolutely in a symmetric interval about its
expansion point, and diverges out side that symmetric interval. The
distance from the expansion point to an endpoint is called the radius of
convergence.

103
Applied Mathematics II

• Any combination of convergence or divergence may occur at the


endpoints of the interval. That is, the series may diverge at both
endpoints, converge at both endpoints, or diverge at one and converge at
the other.
• A power series always converges at the point of expansion.
Example 3. The power series
a 0 + a1 ( x − 5) + a 2 ( x − 5) 2 + a 3 ( x − 5) 3 + L

is expanded around c = 5 . It surely converges at x = 5 , since setting x = 5 gives


a0 + 0 + 0 + L = a0 .

The series converges on an interval which is symmetric about c = 5 . Thus,


− 2 < x < 12 is a possible interval of convergence; 3 < x < 8 is not.
Suppose you know that 3 < x < 7 is the largest open interval on which the series
converges. Then the series can do anything (in terms of convergence or
divergence) at x = 3 and x = 7 . The interval of convergence could be 3 < x < 7
(diverges at both ends), 3 ≤ x ≤ 7 (converges at both ends), or 3 ≤ x < 7 or 3 < x ≤ 7
(converges at one end and diverges at the other).

xn
Example 4. Determine the interval of convergence for the series ∑ 3n .
n =0

Solution: Take the absolute value and apply the Ratio Test:
n +1
x
n +1
bn +1 n +1 x 3n x
lim = lim 3 n = lim n
⋅ n +1
= lim .
n→∞ bn n→∞ x n→∞ x 3 n →∞ 3

3n
x
The series converges for < 1 , that is, for − 3 < x < 3 . It diverges for x < − 3 and for
3
x > 3.
Next let us test the end points separately.
At x = 3 , the series is

3n ∞
∑ 3n = ∑1 =1+ 1 + 1+ L .
n =0 n =0

104
Applied Mathematics II

The series diverges at x = 3 .


At x = − 3 , the series is

(− 3)n ∞
∑ 3 n
= ∑ (− 1) = 1 − 1 + 1 − L .
n

n =0 n=0

The series diverges at x = − 3 .


All together, the series diverges for x ≤ − 3 and for x ≥ 3 . It converges for

− 3 < x < 3 . (Or the series diverges for x ≥ 3 and converges for x < 3 )

You would reach the same conclusion using the Root Test:
1
1  x n n x
lim bn n = lim  n  = .
n→∞ n →∞  3  3
 
x
The Root Test says that series converges for < 1 , that is, for − 3 < x < 3 , and that
3
it diverges for x < − 3 and for x > 3 . The endpoint check is the same as above.

( x − 2 )n .
Example 5. Determine the interval of convergence for the series ∑ n ⋅ 5n
n =1

Solution: Take the absolute value and apply the Ratio Test:
n +1
x−2
n +1
lim
bn +1
= lim
(n + 1)⋅ 5n +1 = lim n

5n x − 2

n→∞ bn n→∞ x−2
n n → ∞ n + 1 5n +1 x−2
n

n ⋅ 5n

1 n 1
= lim x−2= x−2 .
n→ ∞ 5 n + 1 5
1
By the Ratio Test, the series converges (absolutely) for x − 2 < 1 or − 3 < x < 7 .
5
Likewise, the series diverges for x < − 3 or for x > 7 .

1
Check the situation at the endpoints. For x = 7 , the series becomes ∑ n . This is
n =1

the harmonic series, and it diverges.

105
Applied Mathematics II


1
For x = − 3 , the series is ∑ (− 1)n n . This is the alternating harmonic series, and
n =1

it converges by the Alternating Series Test


To summarize, the series converges absolutely for − 3 < x < 7 , converges
conditionally for x = − 3 , and diverges for x < −3 and for x ≥ 7 . The interval of
convergence is − 3 ≤ x < 7 .

( x − 4)
Example 6. For what values of x does the series ∑ n
converge?
n =1

Solution: Let u n =
( x − 4 )n . Then
n

u n +1 n ( x − 4) n +1 1
= n
= x − 4 → x − 4 as n → ∞ .
un (n + 1) ( x − 4) 1
1+
n
By Generalized Ratio Test, the given series is converges, when x − 4 < 1 and

diverges when x − 4 > 1 . Now

x − 4 <1 ⇔ − 1< x − 4 < 1 ⇔ 3< x < 5

so the series converges when 3 < x < 5 and diverges when x < 3 or x > 5 .

The Generalized Ratio Test gives no information when x − 4 = 1 so we must

consider x = 3 and x = 5 separately. If we put x = 3 in the series, it becomes



(−1) n
∑ n , the alternating harmonic series, which converges by Alternating Series
n =1


1
Test. If we put x = 5 in the series, it becomes ∑n , the harmonic series, which
n =1

is convergent.
Therefore the given power series converges for 3 ≤ x < 4 .

Example 7. Determine the interval of convergence for the series ∑ nn xn .
n =0

Solution: Take absolute values and apply the Ratio Test:

106
Applied Mathematics II

bn +1 (n + 1)n +1 x n +1  n +1
n
lim = lim n
= lim (n + 1) x ⋅   .
n→∞ bn n→∞ nn x n →∞  n 
n
 n + 1
We shall compute the last term separately. Let y =   . Then
 n 
n
 n + 1  n + 1  1
ln y = ln  = n ln  = n ln1 +  .
 n   n   n
By L’Höpital’s Rule,
 1
ln1 + 
 1  n
lim ln y = lim n ln1 +  = lim
n →∞ n →∞  n  n→∞ 1
n
 
 
 1   − 1 

 1   n2   
1 +   
 n 1 
lim = lim  =1.
n →∞  1  n →∞  1
− 2  1 + 
 n   n
Hence,
 
 
 lim ln y
 
n → ∞ 
lim y = e = e1 = e .
n →∞

Therefore,
n
 n + 1
lim (n + 1) x ⋅   =+ ∞.
n →∞  n 
This means that the series diverges for all x except x = 0 , the point of
expansion. At x = 0 , the series looks like
∞ ∞
∑ nn 0n = ∑ 0 = 0 ,
n =0 n =0

so it certainly converges.
Activity . Determine the interval of convergence for each of the following series

2 2n x n
a) ∑ n2
.
n =1

107
Applied Mathematics II


( x + 3) n .
b) ∑ n +1
n =0


ln n n
c) ∑ x .
n =3 n

Key terms
• Power series
• Radius of convergence of a power series
• Interval of convergence of a power series
Self Test.
1. Find the radius and interval of convergence of the following series

xn ∞
( x − 4) n ∞
(ln n)( x − 3) n ∞
(2 x − 3) n
a) ∑ 2
n =1 n + 1
b) ∑ (−1)n
n =1 (n + 1) 2
c) ∑
n =1 n
d) ∑
n =1 42n

π n ( x − 1) 2 n
e) ∑
n =1 (2n + 1)!

(n + p)!
2. Find the radius of convergence of the power series ∑ n!(n + q)! x
n =1
n
where p and

q are positive integers.


1
1
3. Prove that if lim cn n = L , then is the radius of convergence of the power
n →∞ L

series ∑c x
n =1
n
n
.

Lessons 3.3&3.4

Lesson Title: Power Series Representation Of Functions,


differentiation and integration of power series
Introduction: In this section we will start talking about how to represent
functions with power series. The natural question of why we might want to do
this will be answered in a couple of sections once we actually learn to do this.

108
Applied Mathematics II

Let’s start off with one that we already know, how to do, although when we first
ran across this series didn’t think of it as a power series nor did we
acknowledge that it represented a function.
Objectives: After a successful accomplishment of this lesson, the student will
be able to:
 Find power series expansions for analytic functions.
 Use power series expansion of some functions to determine power series
expansion of other functions which have some relation.
 Differentiate a given power series and find radius of convergence.
 Integrate a power series and find radius of convergence.

Example 1. Consider the series



∑ x n =1 + x + x 2 + x 3 + L + x n + L
n =0

1
This series represents the function for − 1 < x < 1 . You can see that this is
x −1
reasonable by dividing 1 by 1 − x , or using the formula for the sum of a
geometric series with common ratio r = x .
1
For example, if x = ,
2
1 1 1
= 2 , while 1 + x + x 2 + x 3 + L = 1 + + + L
1− x 2 4
Results on geometric series show that the two expressions are equal.
On the other hand, if x = − 1 ,
1 1
= , while 1 + x + x 2 + x 3 + L = 1 − 1 + 1 − 1 + 1 − L .
1− x 2
The two expressions are not equal; in fact, the series on the right diverges, by
the Divergence Test.

The sum of a series is a function


f ( x) = a 0 + a1 x + a 2 x 2 + L + a n x n + L ,

109
Applied Mathematics II

whose domain is the set of all x for which the series converges. Notice that f
resembles a polynomial. The only difference is that f has infinitely many
terms.
For instance, if we take a n = 1 for all n , the power series becomes the geometric

series

1
∑ x n =1+ x + x 2 + L + x n + L = 1 − x
n =0

Turning this around we can see that we can represent the function
1
f ( x) = ……………………(*)
1− x
with the power series

∑ xn provided that x <1 ………….(**)
n =0

which converges when -1 < x < 1 and diverges when |x| > 1.
This idea of convergence is important here. We will be representing many
functions as power series and it will be important to recognize that the
representations will often only be valid for a range of x’s and that there may be
values of x that we can plug into the function that we can’t plug into the power
series representation.
Let’s extend the relation we get from the above example by undertaking certain
manipulations.

Example 2. Find a power series representation for the following function and
determine it’s interval of convergence.
1
g ( x) =
1+ x 3

Solution: What we want here to do here is to relate this function back to (*).
This is actually easier than it might look. Recall that the x in (*) is simply a
variable and can represent anything. So, a quick rewrite of g (x)

110
Applied Mathematics II

1
gives, g ( x) = and so the − x3 in g (x) holds the same place as the x in (*).
1− ( − x3 )
Therefore, all we need to do is replace the x in (*) and we have got a power
series representation for g (x) .

( )
g ( x) = ∑ − x3
n
provided that − x 3 <1 .
n =0

Notice that we replaced the x in the power series given in the previous example
by − x3 .
All we need to do now is a little simplification.

g ( x) = ∑ (− 1)n x3n provided x <1 ⇒ x <1 .
3

n =0

So, in this case the interval of convergence is the same as the original power
series. This usually won’t happen. More often that not the new interval of
convergence will be different from the original interval of convergence.

Example 3. Find a power series representation for the following function and
determine it’s interval of convergence.

2x2
h( x) = .
1+ x3
Solution: This function is similar to the previous function. The difference is the
numerator and at first glance that looks to be an important difference. Since (*)
doesn’t have an x in the numerator it appears that we can’t relate this function
back to that.
However, we have worked the first example this one is actually very simple
since we can use the result of the answer from that example. To see how to do
this let’s first rewrite the function a little.
1
h( x) = 2 x 2
1+ x3
Now, from the first example we have already got a power series for the second
term so let’s use that to write the function as,

111
Applied Mathematics II


h( x) = 2 x 2 ∑ (− 1)n x 3n provided that x <1 .
n=0

Notice that the presence of x’s outside the series will not affect its convergence
and so the interval of convergence remains the same.
The last step is to bring the coefficient into the series and we will be done.
When we do this make sure and combine the x’s as well. We typically only want
a single x in a power series,

h( x) = ∑ 2 (−1) n x3n + 2 provided x <1 .
n =0

Example 4. Find a power series representation for the following function and
determine it’s interval of convergence.
x
f ( x) =
5− x
Solution: Again we have got an x in the numerator. So, as with the last example
let’s factor that x out and see what we’ve got left.
1
f ( x) = x
5− x
If we had a power series representation for
1
g ( x) =
5− x
we could get a power series representation for f (x) .
So, let’s find one. To use the same thing as above we want the constant term in
the denominator other than the variable to be 1. Thus, it is easy to get
1 1
g ( x) =
5 1− x
5
Now all we need to do to get a power series representation is to replace the x in
x
(**) with . Doing this gives,
5
n
1 ∞ x x
g ( x) = ∑   provided that <1 .
5 n=0 5  5

112
Applied Mathematics II

Now let’s do a little simplification on the series.


n
1 ∞ x
g ( x) = ∑ 
5 n=0 5 

xn
= ∑ n +1
n =0 5

The interval of convergence for this series is,


x 1
<1 ⇒ x <1 ⇒ x < 5 .
5 5
This was the work for the power series representation for g (x) let’s now find a
power series representation for the original function. All we need to do for this
is to multiply the power series representation for g (x) by x and we’ll have it.
1
f ( x) = x
5− x

xn
= x∑ n +1
n=0 5

x n +1

=∑ n +1
n=0 5

The interval of convergence doesn’t change and so it will be x < 5 .

Now we have and idea on how to find the power series representation for some
functions.

In this section we shall use power series to present a wide variety of functions.
A power series may represent a function f (x) , in the sense that whenever the
series converges,, it converges to f (x) . There are two issues here:
1. Where does the series converge?
2. If the series converges at a point, does it converge to f (x) ?

113
Applied Mathematics II

y = f (x)
y = a 0 + a1 x

A(0, f (0))
y = a0

x
O

Fig 2.1

The tangent line y = a 0 + a1 x serves to approximate the function y = f (x) for

small values of x .
We shall concern ourselves with the problem of approximating a given function
y = f (x) …………………………………………...(1)

by a sequence of polynomials f n (x) of the form

f n ( x) = a 0 + a1 x + a 2 x 2 + L + a n x n ……………………..(2)

In particular, since we expect to take larger and larger values of n , we shall be


interested in making the approximation good for small values of x , since we
want the terms that come far out in the series to be small. Hence we first focus
our attention on a portion of the curve y = f (x) in the neighborhood of the point
A(0 , f (0)) in Fig2.1. We then ask the following questions:

Q. What polynomial y = f 0 ( x) = a 0 of degree zero, gives the closest approximation

to the given curve in the immediate neighborhood of the point A ?


A. We want the horizontal line y = a 0 to pass through A . Hence we take

a 0 = f (0) , which is the value of the given function at x = 0 .

Q. What polynomial y = a 0 + a1 x , of degree one, comes closest to fitting the given

curve in the neighborhood of the point A ?

114
Applied Mathematics II

A. The tangent line. That is, we want this approximating line to pass through A
and to have the same slope there that the given curve has. This means that we
should take
a 0 = f (0) and a1 = f ' (0) .

Q. More generally, what polynomial y = a 0 + a1 x + a 2 x 2 + L + a n x n ,of degree n ,

comes closest to fitting the given curve near A ?


A. We take the approximating curve that pass through A and the highest
possible degree of contact with the given curve at A , in the sense that its
derivative of orders one, two, three, . . . , n match the corresponding derivatives
of the given function there. Thus, if we write
f n ( x) = a 0 + a1 x + a 2 x 2 + a3 x 3 + L + a n x n

For the approximating polynomial, then its derivative are


f n ' ( x) = a1 + 2a 2 x + 3a 3 x 2 + L + na n x n −1 ,

f n ' ' ( x) = 2a2 + 3 ⋅ 2a3 x + L + n(n − 1) x n −2 ,

f n ( n ) ( x) = n ! a n .

We substitute x = 0 on the right-hand side of each of the above equations and


set the results, respectively, equal to the values
(n )
f ( 0) , f ' ( 0) , f ' ' ( 0) , K , f ( 0)
of the given function and its first derivatives at x = 0 . We then determine the
coefficients
a 0 , a1 , a 2 , K , a n

of the approximating polynomial y = f n (x) that has the highest degree of contact

with the given curve at A(0 , f (0)) :

115
Applied Mathematics II

a 0 = f (0) ,
a1 = f ' (0) ,
f ' ' (0)
a2 = ,
2!
M
(n)
f (0)
an = .
n!

That is for our approximating polynomial of degree n , we take

f ' ' ( 0) 2 f ' ' ' ( 0) 3 f ( n ) ( 0) n


f n ( x ) = f ( 0) + f ' ( 0) x + x + x +L+ x . ………(3)
2! 3! n!
Using the same argument we can see that, if a function has derivatives of all
orders (it is said to be infinitely differentiable), it is possible to construct a
power series for the function:
(n )
f ' ' (c )
f ( x ) = f (c ) + f ' ( c ) ( x − c ) + ( x − c ) 2 + L + f (c ) ( x − c ) n + L .
2! n!

Example 5. Compute the polynomial f n (x) given by Equation(3) that would be

used to approximate the exponential function f ( x) = e x near x = 0 .


(n )
Solution: We require f (0) , f ' (0) , f ' ' (0) , K , f (0) . For the given function, f ( x) = e x .
Thus
f ( x) = e x ; f ( 0) = e 0 = 1
and
f ' ( x) = e x ; f ' (0) = e 0 = 1,
f ' ' ( x) = e x ; f ' ' (0) = e 0 = 1
M
(n) ( n)
f ( x) = e x ; f (0) = e 0 = 1.
Therefore we have

x2 x3 xn
f n ( x) = 1 + x + + +L+ .
2 ! 3! n!

116
Applied Mathematics II

The question now naturally arises as to whether, for a fixed value of x , our
approximating polynomials f n (x) converge to f (x) as n → ∞ . Since f n (x) of

Equation 3 is the partial sum of the infinite series

f ' ' ( 0) 2 f ' ' ' ( 0) 3 f ( n ) ( 0) n


f n ( x ) = f ( 0) + f ' ( 0) x + x + x +L+ x + L , ……….(5)
2! 3! n!
This question just posed is equivalent to inquiring whether the series (5)
converges to f (x) as sum. Of course, everyone of our approximating
polynomials has the correct value f (0) at x = 0 , we are now interested in
knowing also how far we may go away from x = 0 and still have convergence.
Definition: If f is a function for which there exists constants a 0 , a1 , a 2 , K such

that

f ( x) = ∑ a n ( x − c) n
n =0

for all x in some open interval about c, then we say f is analytic at c. If for some
h>0 the above equality holds for all x in the interval I=(c-h, c+h), then we say f
is analytic on I and we call

∑ a n ( x − c) n
n =0

a power series representation of f on I.

Differentiation and integration of power series


A power series resembles a polynomial; in fact, often it is convenient to think of
a power series as a polynomial of infinite degree. Among the many nice
properties of polynomials is the ease with which they may be differentiated and
integrated. Our next result states that power series may be differentiated and
integrated term by term in the same manner as polynomials. Although we have
the tools to provide justifications for these statements, they are technical and
perhaps best left to a more advanced study.

117
Applied Mathematics II


Since ∑ an x n is a function, we may ask whether its derivative exists. Perhaps
n =0

surprisingly, a power series with a nonzero radius of convergence is always



differentiable; moreover, the derivative is obtained from ∑ an x n by
n =0

differentiating term by term, the way we differentiate polynomials.



Theorem 3.3: Let ∑ an x n be a power series with radius of convergence R > 0 .
n =0


Then ∑ n an x n −1 has the same radius of convergence, and
n =1

d  ∞  ∞ ∞
d
 ∑ an x n  = ∑ n an x n −1 = ∑
dx  n = 0
( )
an x n for x < R .
 n =1 n =1 dx

Proof: It is left as an exercise.


Example 6. Recall that the interval of convergence of

xn
exp( x) = ∑ is (− ∞ , ∞ ) .
n=0 n !

d d ∞ xn
Hence exp ( x) = ∑
dx dx n =0 n !

d  xn 
=∑  
 
n =0 dx  n ! 


n x n −1
=∑
n =1 n !


x n −1
=∑
n =1 (n − 1)!


xn
=∑
n =0 n !

= exp ( x) for all x in ( − ∞ , ∞ ) .


That is the function exp ( x) is its own derivative.

118
Applied Mathematics II


The differentiation theorem states that a power series ∑ an x n with a nonzero
n =0

radius of convergence can be differentiated once. However, because the


derivative is itself a power series with the same radius of convergence, the
derivative also may be differentiated, and thus the original power series can be
differentiated twice. By repeating this process, we conclude that a power series
with radius of convergence R > 0 has derivative of all orders on (− R, R) . The
values of the derivatives of the series at 0 are closely related to the numbers
c0 , c1 , c 2 ,K , as we see in the next theorem. For convenience, we will denote

f (0) by f ( 0) (0) .

Theorem 3.4: suppose a power series ∑ an x n has radius of convergence R > 0 .
n =0

Let

f ( x) = ∑ an x n for − R < x < R …………(1)
n =0

Then f has derivatives of all orders on (− R, R) , and

f ( n ) (0) = n!an for n ≥ 0 ………………..(2)

Consequently,

f ( n ) ( 0)
f ( x) = ∑ for − R < x < R …………..(3)
n =0 n!
Proof: From the discussion preceding this theorem we know that f has
derivatives of all orders on (− R, R) . By substituting x = 0 into (1) we obtain

f (0) (0) = f (0) = a0 = 0!a0

Next we differentiate both sides of (1), using the differentiation theorem:



f ' ( x) = ∑ n an x n −1 …………………..(4)
n =1

By again substituting x = 0 , we obtain


f ' (0) = a1 =1!a1
Differentiating both sides of (4) yields

119
Applied Mathematics II


f ' ' ( x) = ∑ n(n − 1) an x n − 2
n =1

Substituting x = 0 once more, we find that


f ' ' (0) = 2(1)a2 = 2!a2
In the same way performing n differentiations on both sides of (1) and then
substituting x = 0 into the result yields (2). Finally, if we substitute for an from

(2) into (1), we obtain (3).


∞ ∞
Corollary 3.1: Let R > 0 , and suppose ∑ an x n and ∑ bn x n are power series that
n =0 n =0

converges for − R < x < R . If


∞ ∞
∑ an x n = ∑ bn x n for − R < x < R
n=0 n =0

then an = bn for each n ≥ 0 .


∞ ∞
Proof: Let f ( x) = ∑ an x n = ∑ bn x n for − R < x < R . Then, Theorem 3.4 implies that
n =0 n=0

f ( n ) ( 0)
an = = bn .
n!
In particular, the above corollary tells us that two polynomials
an x n + an −1 x n −1 + L + a1 x + a0

and bm x m + bm −1 x m −1 + L + b1 x + b0

which are of course power series having all but a finite number of their
coefficients 0, are the same function if and only if
m = n and b j = a j for 1 ≤ j ≤ n

Suppose that R > 0 and that ∑ an x n converges for − R < x < R . Then we know
n =0


that ∑ an x n is differentiable, and hence continuous, on (− R, R) . It is therefore
n =0


possible to integrate the function ∑ an x n over any closed interval in (− R, R) . The
n =0

120
Applied Mathematics II

next theorem states that we can carry out the integration term by term, just as
we do differentiation.

Theorem 3.5: Let ∑ an x n be a power series with radius of convergence R > 0 .
n =0


 a 
Then ∑  n +n 1  x n +1 has the same radius of convergence, and
n=0

 ∞ n an n +1 ∞  
x ∞ x

∫  n∑=0 n  n∑=0 n + 1
 a t dt = x = ∑  ∫ ant n dt  for x < R .
n=0 0
0 
Proof: It is left as an exercise.
Example 7. We have seen that

1
=1 + x + x 2 + L = ∑ x n for x < 1 .
1− x n=0

Then differentiating and integrating yields


1 d  1  2
=   =1 + 2 x + 3 x + L
(1 − x ) 2
dx  1 − x 

= ∑ n x n −1 for x < 1 , …………………(1)
n =1

and
dx x2 x3 ∞
x n +1 ∞ x n
− ln(1 − x) = ∫ =x+ + +L = ∑ = ∑ + C for x < 1 ……..(2)
1− x 2 3 n = 0 n + 1 n =1 n

But ln(1 − 0) = ln 1 = 0 , so that C = 0 . It is easy to verify that the radii of


convergence of series (1) and (2) are both 1.
Key terms
• Series representation of functions
• Differentiations of power series
• Integration of power series
Self Test.
(−1) n 2 n ∞ ∞
(−1) n 2 n +1
1. Show that both ∑ x and ∑ x converges for all x.
n =1 ( 2 n)! n =1 ( 2n + 1)!

121
Applied Mathematics II


(−1) n 2 n ∞
(−1) n 2 n +1
2. Let f (x) = ∑
n =1 ( 2 n)!
x and g (x) = ∑
n =1 ( 2n + 1)!
x

a) Show that f , ( x) = − g ( x) and g , ( x) = f ( x)

b) Show that f , , ( x) = − f ( x) and g ,, ( x) = − g ( x)


c) What functions do you know that satisfy the properties of (a) and
(b).
arctan t ∞
(−1) n 2 n
3. a) Show that =∑ t for 0 < t < 1 .
t n = 0 2n + 1

arctan t
b) Using part (a) , show that lim =1
t →0 t

cn +1 1
4. Show that if the radius of convergence of ∑c x
n =0
n
n
is R , then lim
n →∞ cn
= .
R

5. Express the following integral as infinite series.

∫ sin x dx ∫e ∫ cos
2 x2
a) b) dx c) x dx

ex − 1
6. Find the power series expansion of cosh x and .
x

Lesson 3.5

Lesson title: Taylor and Maclaurin Series


Definition: Let f be a function with derivatives of all orders throughout some
interval containing c as an interior point. Then the Taylor Series generated by
f at x = c is

f ( n ) (c ) (n )
∑ ( x − c ) n = f (c ) + f ' (c ) ( x − c ) + f ' ' ( c ) ( x − c ) 2 + L + f (c ) ( x − c ) n + L .
n =0 n! 2! n!
The Maclaurin Series generated by f is

f ( n ) ( 0) f ' ' ( 0) 2 f ' ' ' ( 0) 3 f ( n ) ( 0) n
∑ n!
= f ( 0) + f ' ( 0) x +
2!
x +
3!
x +L+
n!
x +L.
n =0

Example 1. Find the Taylor Series for f ( x) = e x about x = 0 .

122
Applied Mathematics II

Solution: This is actually one of the easier Taylor Series that we’ll be asked to
compute. To find the Taylor Series for a function we will need to determine a
general formula for f ( n ) (a ) . This is one of the few where this is easy right from
the start.
To get a formula for f (n ) (0) all we need to do is recognize that,

f ( n ) ( x) = e x n = 0,1, 2 , 3 ,K
and so,
f ( n ) ( 0) = e 0 = 1 n = 0 ,1, 2 , 3 ,K .

Therefore, the Taylor Series for f ( x) = e x is about x = 0



1 n ∞ xn
ex = ∑ x =∑
n =0 n! n =0 n!

Example 2. Find the Taylor Series for f ( x) = e − x about x = 0 .

Solution: There are two ways to do this problem. Both are fairly simple, however
one of them requires significantly less work. We will work both solutions since
the longer one has some nice ideas that we’ll see in other examples.
1. As with the first example we’ll need to get a formula for f (n ) (0) .
However, unlike the first one we’ve got a little more work to do. Let’s
first take derivatives and evaluate them at x=0.
f ( 0) ( x ) = e − x f ( 0 ) ( 0) = 1
f (1) ( x) = − e − x f (1) (0) = − 1
f ( 2) ( x ) = e − x f ( 2 ) ( 0) = 1
f ( 3) ( x ) = − e − x f ( 3 ) ( 0) = − 1
M M
f ( n ) ( x) = (−1) n e − x f ( n ) (0) = (−1) for n =1, 2 , 3 ,K
After a couple of computations we are able to get general formulas for
both f ( n ) ( x) and f ( n ) (0) . We often won’t be able to get a general

formula for f ( n ) ( x) so don’t get too excited about getting that formula.

123
Applied Mathematics II

Also, as we will see it won’t always be easy to get a general formula for
f ( n ) ( a) .
So, in this case we’ve got general formula so we need to do is plug this
into the Taylor Series formula and be done with the problem.
(−1) n x n

e =∑
−x

n=0 n!
2. The previous solution wasn’t too bad and we often have to do things in
that manner. However, in this case there is a much shorter solution
method. In the previous section we used series that we’ve already
found to help us find a new series. Let’s do the same thing with this
one. We already know a Taylor Series for e x about x = 0 and in this
case the only difference is we’ve got a “-x” in the exponent instead of
just an x.
So, all we need to do is replace the x in the Taylor Series that we found
in the first example with “-x”.

e− x = ∑

(− x )n = ∞
(− 1)n x n
n!
∑ n!
n =0 n =0

This is a much shorter method of arriving at the same answer so don’t forget
about using previously computed series where possible.
2
Example 3. Find the Taylor Series for f ( x) = x 4 e−3 x about x = 0 .
Solution: For this example we will take advantage of the fact that we already have
a Taylor Series for e x about x = 0 . In this example, unlike the previous example,
doing this directly would be significantly longer and more difficult.
( −3 x 2 ) n

4 −3 x 2
x e =x ∑ 4

n=0 n!

(−3) n x 2 n
= x4 ∑
n =0 n!
(−3) n x 2 n + 4

=∑
n=0 n!

124
Applied Mathematics II

To this point we’ve looked at Taylor Series about x=0 (also known as Maclaurin
Series) so let’s take a look at a Taylor Series that isn’t about x=0. Also, we’ll pick
on the exponential function one more time since it makes some of the work
easier. This will be the final Taylor Series for exponentials in this section.
Example 4. Find the Taylor Series for f ( x) = e − x about x = 4 .

Solution: Finding a general formula for f ( n ) (−4) is fairly simple.

f ( n ) ( x) = (−1) n e− x f ( n ) (−4) = (−1) n e 4


The Taylor Series is then,

(−1) e 4
e =∑
−x
( x + 4) n .
n =0 n!
Next, Let’s work on some examples that do not involve the exponential function.
Example 5. Find the Maclaurin Series for f ( x) = cos( x) .
Solution: First we shall take some derivatives of the function and evaluate them
at x=0.
f ( 0) ( x) = cos x f ( 0 ) ( 0) = 1
f (1) ( x) = − sin x f ( 0 ) ( 0) = 0
f ( 2) ( x) = − cos x f ( 2 ) ( 0) = − 1
f (3) ( x) = sin x f ( 3 ) ( 0) = 0
f ( 4) ( x) = cos x f ( 4 ) ( 0) = 1
f (5) ( x) = − sin x f ( 5 ) ( 0) = 0
M M
In this example, unlike the previous ones, there is not an easy formula for
either the general derivative or the value of the derivative at x=0. However, there
is a clear pattern for the values of the derivatives. So, let’s plug what we’ve got
into the Taylor Series and see what we get,

125
Applied Mathematics II


f ( n ) ( 0) n
cos x = ∑ x
n=0 n!
f ' ' ( 0) 2 f ' ' ' ( 0) 3 f ( 4 ) ( 0) 4 f ( 5 ) ( 0) 5
= f ( 0) + f ' ( 0) x + x + x + x + x +L
2! 3! 4! 5!
1 2 1 1
=1 + 0 − x + 0 + x 4 + 0 − x6 +L
2! 4! 6!
So, we only pick up terms with even powers on the x’s. This doesn’t really help
us to get a general formula for the Taylor Series. However, let’s drop the zeros
and “renumber the terms as follows to see what we can get.
1 2 1 4 1 6
cos x = {
1 − x + x − x +L
2! 4! 6!
n=0 123 123 123
n =1 n=2 n =3

By renumbering the terms w can actually reach at the following formula for the
Taylor Series of the given function

(−1) x 2 n
cos x = ∑
n = 0 ( 2n ) !

Activity.
Find the Taylor series of f about the given point a .
1
a) f ( x) = , a = −1 b) f ( x) = ln x; a = 3 c) f ( x) = x , a =1
x
d) f ( x) = ln 3 x; a = 1 e) f ( x) = sin 3 x; a = 0
Key terms
• Taylor series
• Maclaurin series
Self Test.
1. Find the fourth Taylor polynomial of the given function about a.
π
a) f ( x) = x 4 − x + 2; a = −1 b) k ( x) = arctan x; a = c) f ( x) = tan x; a = 1
3
2. Find the Maclurin series of f where
x −1
a) f ( x) = cos 2 4 x b) f ( x) = e x cos x c) f ( x) =
x +1

126
Applied Mathematics II

− 3x + 2
d) f ( x) = e) f ( x) = sinh 3 x
2 x 2 − 3x + 1
3. Let f ( x) = tan x . Using the fact that f (0) = 0 and f . ( x) = 1 + [ f ( x)]2 , find the
sum of the first six terms in the Taylor series of f about 0.

Lesson 3.6&3.7

Lesson title: Taylor’s Formula, Applications


Introduction: Recall that we were able to express some functions using power
series. Based on this fact we will see approximation of functions using Taylor
series expansions. Many of the applications of series, especially those in
differential equation fields, rely on the fact that functions can be represented as
a series.

Objectives: After a successful accomplishment of this section the student will


be able to:
 Determine nth degree Taylor polynomial for analytic functions.
 Use Taylor’s Formula to approximate functions.
 Find series solutions of some differential equations.

Approximation By Taylor’s Formula


In the previous section we were able to express some functions f as the sum of
their Taylor series:

f ( n ) (c )
f ( x) = ∑ n!
(x − c )n .
n =0

As with any series, the partial sums are approximations to the total sum of the
series. In the case of the Taylor series the partial sums are
n
f ( k ) (c )
Tn ( x) = ∑ (x − c )k
k =0 k!
( n)
f ' (c )
= f (c ) + (x − c )+ f ' ' (c) (x − c )2 +L + f (c) (x − c )n
1! 2! n!

127
Applied Mathematics II

Notice that Tn is a polynomial of degree n called the nth-degree Taylor

polynomial of f at c.

If f is the sum of its Taylor series, then Tn ( x) → f ( x) as n → ∞ and so Tn can be

used to approximate f : f ( x) ≈ Tn ( x) . It is useful to be able to approximate a

function by a polynomial because polynomials are the simplest functions. We


can gain information about a function by looking at its Taylor polynomials Tn .

Example 1. From an example in the previous section we know that the Taylor
series for f ( x) = e x at c = 0 is

x x 2 x3 xn
1+ + + + L + + L
1! 2! 3! n!
So its first three Taylor polynomials at 0 (or Maclaurin polynomials) are

x2 x2 x3
T1 ( x) =1 + x T2 ( x) =1+ x + T3 ( x) =1+ x + +
2! 2! 3!
Example 2. For the function f (x) = cos(x) plot the function as well as
T2 ( x) , T4 ( x) , and T8 ( x) on the same graph for the interval [-4,4].

Solution:
Here is the general formula for the Taylor polynomials for cosine.
n
(−1) k x 2 k
Tn ( x) = ∑
k = 0 ( 2k )!

The three Taylor polynomials that we’ve got are then,

x2
T2 ( x) =1 −
2
x2 x4
T4 ( x) =1 − +
2 24
x 2 x 4 x6 x8
T8 ( x) =1 − + − +
2 24 720 40320
Here is the graph of these three Taylor polynomials as well as the graph of
cosine.

128
Applied Mathematics II

As we can see from this graph as we increase the degree of the Taylor
polynomial it starts to look more and more like the function itself. In fact by the
time we get to T8 ( x) the only difference is right at the ends. The higher the

degree of the Taylor polynomial the better it approximates the function.


Also the larger the interval the higher degree Taylor polynomial we need to get a
good approximation for the whole interval.
Theorem 3.6 (Taylor’s Formula): If f has n+1 derivatives in an interval I that
contains the number c then for x in I there is a number z strictly between x
and c such that

f ' (c ) f ' ' (c ) f ( n ) (c )


f ( x ) = f (c ) + ( x − c) + ( x − c) 2 + L + ( x − c) n + Rn ( x)
1! 2! n!

f ( n +1) ( z )
Where Rn ( x) = ( x − c) n +1 .
(n + 1)!
Note 1: For special case n=0, if we put x=b, c=a, and z=c, we get
f (b) = f (a ) + f ' (c) (b − a ) ,
which is the Mean Value Theorem.
Note 2: We can write the Taylor Formula using Taylor polynomials as
f ( x) = Tn ( x) + Rn ( x)

129
Applied Mathematics II

Notice that the remainder term

f ( n +1) ( z )
Rn ( x) = ( x − c) n +1
(n + 1)!

is very similar to the terms in the Taylor series except that f ( n +1) is evaluated at
z instead of at c. All we can say about z is that it lies somewhere between x and
c. The expression for Rn (x) in equation (*) is known as Lagrange’s form of the

remainder term.
The error in the approximation f ( x) ≈ Tn ( x) is

f ( x) − Tn ( x) = Rn ( x)

And this can often be estimated by using equation (*).


Example 3. The Maclaurin series for ln(1 + x) is

x 2 x3 x 4
ln(1 + x) = x − + − +L .
2 3 4
What is the largest error which might result from using the first three terms of
the series to approximate ln(1 + x), if 0 ≤ x ≤1 ?
Solution: The remainder term is

f ( n +1) (c) n +1
Rn ( x) = x
(n +1)!
where 0 < c < x. Note that since x ≤1 we have 0 < c < 1. We want to estimate the

maximum size of R3 ( x) . (I take absolute values, because I only care about the

magnitude of the error.)

(We used the fact that x ≤1 .)

130
Applied Mathematics II

1 1
How large can 4
be, given that 0 < c < 1? As c goes from 0 to 1,
(1 + c) (1 + c) 4
1
decreases, so it is never larger than it would be if c = 0. In that case, =1 .
(1 + c) 4
Therefore,
1 1
R3 ( x) ≤ ⋅ 1= .
4 4
1
The error is no greater than .
4

When using a Taylor polynomial Tn to approximate a function f , we have to

ask the question: How good an approximation is it? Or, how large should we
take in order to achieve the desired accuracy

Activity:
1.Use Taylor polynomials to approximate the number with an error less than
0.001
−1
3
π 4
a) 95 b) e c) sin d) 17
5

2. Find the second Taylor polynomial of f ( x) = 1 + x 4 about 0.

Using Power series to solve ordinary differential equations


Consider the second order linear equation
y ' ' + a ( x) y ' + b( x) y = f ( x)
If the functions a and b are not constant functions, then there is, in general, no
way to obtain a closed form solution to the equation even in the homogeneous
case ( f = 0 ). In this section we show how power series can be used to obtain
series solutions to the equation above in some cases. The examples we present

131
Applied Mathematics II

are merely illustrative of a technique that sometimes works. For a more


complete discussion you should refer a book on differential equations.
The fundamental assumption used in solving a differential equation by power
series method is that the solution of the differential equation can be expressed
in the form of power series, say,

y = ∑ cn x n ………………(1)
n=0

Once this assumption has been made, power series expansions for y ' , y ' ' ,K can
be obtained by differentiating equation (1) term by term:

y ' = ∑ ncn x n −1 , …………………………(2)
n =1


y ' ' = ∑ n(n − 1)cn x n − 2 , etc.,…………….(3)
n=2

And this can then be substituted into the given differential equation. After all
the indicated operations have been carried out, and like powers of x have been
collected, we obtain an the expression of the form

k0 + k1 x + k2 x 2 + L = ∑ kn x n = 0 ……………..(4)
n =0

where the coefficients k0 , k1 , k2 ,K are expressions involving the unknown

coefficients c0 , c1 , c2 ,K . Since equation (4) must hold for all values of x in some

interval, all the coefficients k0 , k1 , k2 ,K must vanish. From the equations

k0 = 0 , k1 = 0 , k2 = 0 , K

it is possible to determine successively the coefficients c0 , c1 , c2 ,K . In this

section we will illustrate this procedure by means of several examples, without


concerning ourselves with questions of the convergence of the power series
under consideration or the inherent limitations of the method. We will see that
power series provides a powerful method for solving certain linear differential
equations with variable coefficients. First, however, in order to check that the

132
Applied Mathematics II

power series method provide the required solution, we shall solve some
problems that could be solved more easily by other methods.
Example 4. Consider the initial value problem
y ' = y + x2 , y ( 0) = 1
Solution: Inserting equation (1) and (2) into the equation, we have
c1 + 2c2 x + 3c3 x 2 + 4c4 x3 + L = (c0 + c1x + c2 x 2 + c3 x3 + L) + x 2 .

Collecting like powers of x, we obtain


(c − c0 ) + (2c2 − c1 ) x + (3c3 − c2 − 1) x 2 + (4c4 − c3 ) x 3 + L = 0 .

Equating each of the coefficients to zero, we obtain the identities


c1 − c0 = 0 , 2c2 − c1 = 0 , 3c3 − c2 − 1= 0 , 4c4 − c3 = 0 , K ,

from which we find that


c1 c0 c +1 c + 2 c c +2
c0 = c1 , c2 = = , c3 = 2 = 0 , c4 = 3 = 0 , K.
2 2! 3 3! 4 4!
With these values, equation (1) becomes
c0 2 c0 + 2 3 c0 + 2 4 c0 + 2 5
y = c0 + c0 x + x + x + x + x +L
2! 3! 4! 5!
 x 2 x3 x4   x2 
= (c0 + 2) 1+ x + + + + L − 2 1+ x + .
 2! 3! 4!   2! 

Looking carefully at the series in parenthesis, we recognize the expansion for


e x , so we have the general solution

y = (c0 + 2) e x − x 2 − 2 x − 2 .

To solve the initial value problem, we set x=0 to obtain


1 = y (0) = c0 + 2 − 2 = c0 .

Thus the solution of the initial value problem is given by the equation
y = 3e x − x 2 − 2 x − 2 .
Example 5. Solve
y '' + y = 0 .
Solution: Using equations (1) and (2) above, we have
(2c 2 )( )
+ 3 ⋅ 2 c3 x + 4 ⋅ 3c4 x 2 + L + c0 + c1x + c2 x 2 + L = 0 .

133
Applied Mathematics II

Gathering like powers of x yields


( 2 c 2 + c 0 ) + (3 ⋅ 2 c 3 + c 1 ) x + ( 4 ⋅ 3c 4 + c 2 ) x 2 + L = 0 .

Setting each of the coefficients to zero, we obtain


2c2 + c0 = 0 , 3 ⋅ 2c3 + c1 = 0 , 4 ⋅ 3c4 + c2 = 0 , 5 ⋅ 4c5 + c3 = 0 , K , and

c0 c1 c2 c0 c3 c1
c2 = − , c3 = − , c4 = − = , c5 = − = ,K.
2! 3! 4 ⋅ 3 4! 5 ⋅ 4 5!
Substituting these values into the power series (1) for y yields
c0 2 c1 3 c0 4 c1 5
y = c0 + c1x − x − x + x + x +L .
2! 3! 4! 5!
Splitting this series into two parts, we have
 x2 x4   x3 x 5 
y = c0 1− + − L + c1  x − + −L
 2! 4!   3! 5! 
This is the familiar general solution;
y = c0 cos x + c1 sin x .

We observe that the power series method produces two arbitrary solutions c0 ,c1

which therefore yields the general solution of the given differential equation.
Activity 3.
Find a non zero series solution of the following differential equations.
dy d2y d2y dy
a) − 5y = 0 b) x 2 − 6 y = 0; y (1) = 5 c) x 2 + x − 49 y = 0
dx dx 2 dx 2
dx
Key terms
• Taylor formula
• Using Taylor series in solving ordinary differential equations
Self Test.
x

1. Let f ( x) = e 2 ; a = 0, n = 4;[−1,0] . Then find

a) T4 ( x) at x=a
b) R4 ( x) at x=a

134
Applied Mathematics II

c) an upper bound on the absolute value of the error if f (x) is


approximated over the given interval by the Taylor polynomial obtained in
part (a).

k n xn
2. Show that y = ∑ satisfies y , − ky = 0 for any fixed k .
n =1 n!

3. Find the general series solution of the equation y '' − 3xy ' − 3 y = 0

d2y dy 2
4. Find the particular series solution of x 2
− x + y = 0 that satisfies the
dx dx
initial conditions y (0) = 0 and y ' (0) = −7 .

Summary
• A power series in ( x − c) is a series of the form

∑ an ( x − c)n = a0 + a1 ( x − c) + a2 ( x − c)2 +L
n=0

This is referred to as a power series about c or center at c. Here we


adopt the assumption that ( x − c) 0 = 1 even when x=c.
i) A power series in x is said to converge at x if the series of real

numbers ∑ an x n converges. Otherwise it is said to diverge at x
n=0

ii) A power series is said to converge in a set D of real numbers if it


converges for every real number x in D.


• For a given power series ∑ an ( x − c) there are only three possibilities:
n=0

4. The series converges only when x = c .


5. The series converges for all x .
6. There is a positive number R such that the series converges if
x − c < R and diverges if x − c > R . At x − c = R , the series may

converge or diverge.

135
Applied Mathematics II

a. The set of values of x for which the power series converges is called the
interval of convergence of the power series

b. Let ∑ an x n be a power series with radius of convergence R > 0 . Then
n=0

∑ n an x n −1 has the same radius of convergence, and
n =1

d  ∞  ∞ ∞
d
 ∑ an x n  = ∑ n an x n −1 = ∑
dx  n = 0
(
dx
)
an x n for x < R .
 n =1 n =1


• Let ∑ an x n be a power series with radius of convergence R > 0 . Then
n=0


 a 
∑  n +n 1  x n +1 has the same radius of convergence, and
n=0

 ∞ n an n +1 ∞  
x ∞ x

∫  n∑=0 n  n∑=0 n + 1
 a t dt = x = ∑  ∫ ant n dt  for x < R .
n=0 0
0 
• Let f be a function with derivatives of all orders throughout some interval
containing c as an interior point. Then the Taylor Series generated by f at
x = c is

f ( n ) (c ) f ' ' (c) f ( n ) (c )
∑ n!
( x − c ) = f (c ) + f ' (c ) ( x − c ) +
n
2!
(x − c ) + L +
2
n!
( x − c )n + L .
n =0

The Maclaurin Series generated by f is



f ( n ) ( 0) f ' ' ( 0) 2 f ' ' ' ( 0) 3 f ( n ) ( 0) n
∑ n!
= f ( 0) + f ' ( 0) x +
2!
x +
3!
x +L+
n!
x +L.
n =0

• If w express the function as the sum of their Taylor series:



f ( n ) (c )
f ( x) = ∑ n!
(x − c )n , then , as with any series, the partial sums are
n =0

approximations to the total sum of the series. In the case of the Taylor
series the partial sums are

136
Applied Mathematics II

n
f ( k ) (c )
Tn ( x) = ∑ (x − c )k
k =0 k!
( n)
f ' (c )
= f (c ) + (x − c )+ f ' ' (c) (x − c )2 +L + f (c) (x − c )n
1! 2! n!

Notice that Tn is a polynomial of degree n called the nth-degree Taylor

polynomial of f at c. This can be used as an approximation of the


above series.
• If f has n+1 derivatives in an interval I that contains the number c then
for x in I there is a number z strictly between x and c such that

f ' (c ) f ' ' (c ) 2 f ( n ) (c )


f ( x ) = f (c ) + ( x − c) + ( x − c) + L + ( x − c) n + Rn ( x)
1! 2! n!

f ( n +1) ( z )
where Rn ( x) = ( x − c) n +1 .
(n + 1)!

• Power series can be used to solve an ordinary Differential equation of the


form: y ' ' + a ( x) y ' + b( x) y = f ( x) .

Review exercise:
1. Find the limit of the following sequence.
n
 e n  (2n − 1)π 
a) lim 1 +  b) lim ( n 2 + n − n 2 − n ) c) lim sin  
n →∞
 n n →∞ n →∞
 2 
n
 13 23 n3 
d) lim  4 + 4 + ..... + 4 
n →∞ n n n 

2. Suppose the number of bacteria in a culture is growing exponentially,
with a doubling time of 10 hours. Suppose also there are initially 100

137
Applied Mathematics II

bacteria in the culture. Find a formula for the an of bacteria in the

culture after n hours.


an + 1
3. Suppose that a1 = 1 , a2 = 1 and an +1 = an + an −1 for n ≥ 2 . Now let bn = for
an

1+ 5
n ≥ 1 . Then show that lim bn = .
n →∞ 2
 nn 
4. a) Show that  n  is decreasing sequence.
 n!e 
n − 2
b) Show that   is increasing sequence.
n + 2

5. Determine whether the series converges or diverges.

n

1 ∞
(ln n) 4 ∞
2n + 1 ∞
 nn 
a) ∑
n =1 ( n + n n )
b) ∑
n =1 n
c) ∑ 2
n =1 3n + n
d) ∑  
n =1  n! 


(2n)! ∞
6n ∞
3n n!
e) ∑ n f) ∑ 2 2
g) ∑
n =1 2 n! n =1 n (ln n ) n =1 ( 2n)!

6. Find the value of p for which the following series converge.



πn ∞
1 ∞
1
a) ∑p
n =1
2n
b) ∑
n =1 ln p
n
c) ∑ (ln p)
n =1
n

7. Find the sum of the series (if it conveges)



2n + 3n ∞
 1 ∞
a) ∑
n =1 6
n +1
b) ∑
n =1
ln1 + 
 n
c) ∑(
n =1
n − n + 1)

8. Express the repeating decimal 27.1318318318….. as a fracrion.



2n + 3
9. Show that ∑ (−1)
n =1
n

(n + 1)(n + 2)
=1


x
10. Show that (1 − x)∑ nx n = for x < 1
n =1 1− x
11. Find the interval and radius of convergence of the following series.

138
Applied Mathematics II


(ln n) 2 n ∞
3n 3n ∞
(n!) 2 n ∞
n2n
a) ∑ (−1)n
n =1 n2
x b) ∑
n =1 5
2n
x c) ∑(
n =1 (2n)!
x d) ∑
n =1 ( 2n)!


( x − 1) n ∞
n 2 ( x + 2) n ∞
(1 − x) 2 n
e) ∑
n =1 n n
g) ∑
n =1 (n + 1)!
h) ∑
n =1 4n n
1 − cos 3 x 1 − cos 3 x
12. Find a power series for 2
and use it to evaluate lim
x x →0 x2
π
10 Find the third Taylor polynomial of secx about
6
11 Let f ( x) = x 6 − 3x 4 + 2 x − 1
a) Find the fifth Taylor polynomial of f about 0.
b) Find the fourth Taylor polynomial of f about -1.
c) Find the Taylor series of f about -1.

2
12 Suppose ∑a
n =1
n is a series such that the nth partial sum sn = 2 −
n
for each

positive integer n.
100
a) Find ∑a
n =1
n

b) Determine whether the series converges or diverges and if it converges,


find its sum.
c) Find a formula for an in terms of n.
1
4
1
13 Estimate ∫
0
3
dx with an error less than 0.001
(1 + x ) 2

∞ ∞
14 Prove that if the power series ∑ an x n and
n =1
∑b x
n =1
n
n
have the same sum on

an interval (-r, r), then an = bn .

Unit Assessment:

• Assignment for submission


• Quiz

139
Applied Mathematics II

• Active participation in group discussion


• Individual and group presentation
• Mid examination
• Final examination

Reference:
1. Howard Anton,(1980), Calculus with Analytic Geometry
2. James Starwart, third edition, Calculus with analytic geometry.
3. Larson R.E and Hoster R.P,(1979), Calculus with Analytic geometry
4. Satsty S.S, (2001), second edition, Engineering Mathematics.
5. Sherman K. Stein and Anthony Barcellos, Fifth Edition.Calculus
and analytic geometry.

140

Potrebbero piacerti anche